You are on page 1of 65

Total Marks : 200

Online Prelims TEST - 25 (SUBJECT)


( InsightsIAS Mock Test Series for UPSC Preliminary Exam 2020 ) Mark Scored : 31.33

1 The tributaries of river Barak are


1. Makku
2. Trang
3. Sonai

Which of the statements given above is/are correct?


A. 1 only
B. 2 and 3 only
C. 1 and 3 only
D. 1, 2 and 3

Your Answer :
Correct Answer : D

Answer Justification :
3
2 71
The Barak river, the head stream of Meghna, rises in the hills in Manipur. The important tributaries

8 25
of the river are Makku, Trang, Tuivai, Jiri, Sonai, Rukni, Katakhal, Dhaleswari, Langachini,
Maduva and Jatinga. Barak continues in Bangladesh till the combined
8 63 Ganga-Brahmaputra join it
near Bhairab Bazar. -
o m
il.c
ma of India
2 Consider the following statements regarding State Emblem
g
0 5@ofat India.
1. It is an adaptation of the Lion Capital of Ashoka Sarnath.

26has a bull on the right and a galloping horse on the left.


2. The bell-shaped lotus is a part of State Emblem
5 1
3. In the State Emblem of India, the abacus

m r7
- above is/are correct?
Which of the statements given
y
A. 1 and 2 only R o
B. 2 and 3 onlyj oy
i
C. 1 and t u3nonly
D.M1,r2 and 3

Your Answer : C
Correct Answer : C

Answer Justification :

The State Emblem of India is an adaptation of the Lion Capital of Ashoka at Sarnath. In the
original, the Lion Capital has four lions mounted back to back, on a circular abacus. The frieze of
the abacus is adorned with sculptures in high relief of an elephant, a galloping horse, a bull and a
lion separated by intervening Dharma Chakras. The abacus rests on a bell-shaped lotus. The
profile of the Lion Capital showing three lions mounted on the abacus with a Dharma
Chakra in the centre, a bull on the right and a galloping horse on the left, and outlines of
Dharma Chakras on the extreme right and left was adopted as the State Emblem of India
on January 26, 1950. The bell-shaped lotus was omitted. Hence statement 2 is incorrect.
The motto “Satyameva Jayate”-Truth alone Triumphs —written in Devanagari script below the
profile of the Lion Capital is part of the State Emblem of India. In the State Emblem lies the official

prelims.insightsonindia.com 1
© Insights Active Learning | All rights reserved - 156493. You may not reproduce, distribute or exploit the contents in any form without
written permission by copyright owner. Copyright infringers may face civil and criminal liability
Total Marks : 200
Online Prelims TEST - 25 (SUBJECT)
( InsightsIAS Mock Test Series for UPSC Preliminary Exam 2020 ) Mark Scored : 31.33

seal of the Government of India. Its use is regulated by the State Emblem of India (Prohibition of
Improper Use) Act, 2005 and The State Emblem of India (Regulation of Use) Rules, 2007

3 Consider the following statements regarding Solicitor General of India


1. The post of Solicitor General of India is statutory in nature.
2. Solicitor General is appointed by President of India on the recommendation of Attorney-General.

Which of the statements given above is/are correct?


A. 1 only
B. 2 only
C. Both 1 and 2
D. Neither 1 nor 2

Your Answer : B
Correct Answer : A
1 3
Answer Justification : 5 27
2
6 38
-8
Solicitor General of India
m
i l .co advisor, and its primary lawyer in the
The Solicitor General of India is the government’s chief legal
Supreme Court of India. The Solicitor General of India
mbya is the secondary law officer of the country,
@ g
assists the Attorney-General, and is himself assisted several Additional Solicitors General of
5 Solicitor General and the Additional Solicitors
India. Like the Attorney-General for India,0the
6
12 on behalf of the Union of India in terms of the Law
General advise the government and appear
5
7 1972.
Officers (Terms and Conditions)rRules,

However, unlike the opost


-m
y of Attorney-General for India, which is a Constitutional post
R
under Article 76,ythe posts of the Solicitor General and the Additional Solicitors General
u njo
are merely statutory. Appointments Committee of the Cabinet appoints the Solicitor
t
riHence statement 2 is incorrect.
General.
M
4 Consider the following statements regarding Kendriya Hindi Samiti
1. It is chaired by Home Minister
2. It is the apex policy making body which lays the guidelines for the propagation and progressive use
of Hindi as official language of the Union.

Which of the statements given above is/are correct?


A. 1 only
B. 2 only
C. Both 1 and 2
D. Neither 1 nor 2

Your Answer : B
Correct Answer : B

Answer Justification :

prelims.insightsonindia.com 2
© Insights Active Learning | All rights reserved - 156493. You may not reproduce, distribute or exploit the contents in any form without
written permission by copyright owner. Copyright infringers may face civil and criminal liability
Total Marks : 200
Online Prelims TEST - 25 (SUBJECT)
( InsightsIAS Mock Test Series for UPSC Preliminary Exam 2020 ) Mark Scored : 31.33

The Kendriya Hindi Samiti was constituted in 1967. Chaired by Prime Minister it is the apex
policy making body which lays the guidelines for the propagation and progressive use of
Hindi as official language of the Union. Hence statement 1 is incorrect. The Committee of
Parliament on official language was constituted in 1976 to periodically review the progress in the
use of Hindi as the official language of the Union and to submit a report to the President.

5 Consider the following statements regarding Municipalities


1. The first Municipal Corporation was set-up in the former Presidency Town of Bombay.
2. Local government is Entry 5 of the concurrent List under 7th schedule of the constitution.

Which of the statements given above is/are correct?


A. 1 only
B. 2 only
C. Both 1 and 2
D. Neither 1 nor 2
1 3
Your Answer : D 5 27
2
Correct Answer : D
6 38
-8
Answer Justification :
o m
il.c
Municipalities
gma
@
05India. The first such Municipal Corporation was
Municipal bodies have a long history in
6
12 of Madras in 1688; and later in Bombay and Kolkata
set-up in the former Presidency Town
5
r7
in 1726. Hence statement 1 is incorrect.
-m
oyhas made detailed provisions for ensuring protection of democracy in
The Constitution of India
R
Parliament and in ythe state legislatures. However, the Constitution did not make the local self-
government in jo areas a clear-cut constitutional obligation.
nurban
rit u
M
While the Directive Principles of State Policy refer to Village Panchayats, there is no specific
reference to municipalities except the implicity in Entry 5 of the State List, which places
the subject of local self-governments as a responsibility of the states. Hence statement 2 is
incorrect.

6 Consider the following statements regarding Pradhan Mantri Krishi Sinchayee Yojana
1. PMKSY has been conceived by amalgamating Accelerated Irrigation Benefit Programme (AIBP),
Integrated Watershed Management Programme (IWMP) and the On-Farm Water Management
(OFWM).
2. The programme will be supervised and monitored by an Inter-Ministerial National Steering
Committee (NSC) will be constituted under the Chairmanship of Minister for Agriculture and farmer
welfare.

Which of the statements given above is/are correct?


A. 1 only
B. 2 only

prelims.insightsonindia.com 3
© Insights Active Learning | All rights reserved - 156493. You may not reproduce, distribute or exploit the contents in any form without
written permission by copyright owner. Copyright infringers may face civil and criminal liability
Total Marks : 200
Online Prelims TEST - 25 (SUBJECT)
( InsightsIAS Mock Test Series for UPSC Preliminary Exam 2020 ) Mark Scored : 31.33

C. Both 1 and 2
D. Neither 1 nor 2

Your Answer :
Correct Answer : A

Answer Justification :

Pradhan Mantri Krishi Sinchayee Yojana (PMKSY)

The scheme has been approved with an outlay of Rs. 50,000 crore for a period of 5 years (2015-16
to 2019-20). The major objective of PMKSY is to achieve convergence of investments in
irrigation at the field level, expand cultivable area under assured irrigation, improve on-
farm water use efficiency to reduce wastage of water, enhance the adoption of precision
irrigation and other water saving technologies (Per drop, More crop), promote sustainable
water conservation practices etc. Cabinet decision was taken in July, 2016 for implementation of
13 River
PMKSY in a mission mode. The mission is administered by Ministry of Water Resources,
7
Development and Ganga Rejuvenation with the Per Drop More Crop component
2 52 being administered
8
by Department of Agriculture, Cooperation and Farmers Welfare (DAC&FW).
3
6
-8
PMKSY has been conceived amalgamating ongoing schemes viz. Accelerated Irrigation
Benefit Programme (AIBP) of the Ministry of Water.c om
Resources, River Development &
Ganga Rejuvenation (MoWR,RD&GR), Integrated i l
a Watershed Management Programme
g m
(IWMP) of Department of Land Resources (DoLR) and the On Farm Water Management
5@Cooperation (DAC).
(OFWM) of Department of Agriculture0and
1 26
The programme will be supervised
r 75 and monitored by an Inter-Ministerial National
Steering Committee (NSC)
y - mwill be constituted under the Chairmanship of Prime Minister
o concerned Ministries. Hence statement 2 is incorrect.
with Union Ministers from
R
j oy
u n
A National Executive Committee (NEC) will be constituted under the Chairmanship of Vice
t
i NITI Aayog to oversee programme implementation, allocation of resources, inter-
Mr coordination, monitoring & performance assessment, addressing administrative issues
Chairman,
ministerial
etc.

7 Consider the following statements regarding Livestock Census


1. The first livestock census was conducted during 1919-1920
2. It is being conducted for every five years by all states/UTs of the country.

Which of the statement above is/are correct?


A. 1 only
B. 2 only
C. Both 1 and 2
D. Neither 1 nor 2

Your Answer :
Correct Answer : C

prelims.insightsonindia.com 4
© Insights Active Learning | All rights reserved - 156493. You may not reproduce, distribute or exploit the contents in any form without
written permission by copyright owner. Copyright infringers may face civil and criminal liability
Total Marks : 200
Online Prelims TEST - 25 (SUBJECT)
( InsightsIAS Mock Test Series for UPSC Preliminary Exam 2020 ) Mark Scored : 31.33

Answer Justification :

Livestock Census

The first livestock census was conducted during 1919-1920 and since then it is being
conducted every five years by all states/UTs of the country. It is the only source, which gives
disaggregated information on various species of farm animals and poultry birds. The 19th Livestock
Census was conducted in 2012 in the country in participation with animal husbandry departments
of the states/UTs. Breed Survey was also undertaken in 2013 in order to estimate the breed wise
number of livestock population on a general principle of 15 per cent sample village across the
country.

8 Consider the following statements regarding Lalit Kala Akademi


1. It is an autonomous body, which is fully funded by the Ministry of Culture.
2. It was set up in 1954 as government’s apex cultural body in the field of visual arts in India.

1 3
Which of the statements given above is/are correct?
5 27
A. 1 only 2
B. 2 only 6 38
C. Both 1 and 2 -8
o m
il.c
D. Neither 1 nor 2

Your Answer : gma


5 @
Correct Answer : C 0
126
5
Answer Justification : r7
-m
Lalit Kala Akademi, the
R oy National Academy of Art, was set up in 1954. Lalit Kala Akademi is
joy which is fully funded by the Ministry of Culture. The Akademi is an
the government’s apex cultural body in the field of visual arts in India. It is an
autonomousnbody,
u has rendered service to the nation in the arts sphere long before the world woke up
itthat
r
institution
to theM global impact of Indian art. It has established, preserved and documented a permanent
collection that reflects the vitality, complexity and unfolding patterns of modern and contemporary
art in India.

All through the year it presents exhibitions and educational programmes of unparalleled
significance; sustains a library, art collection, archives, conservation laboratory and supports
scholars and publications of pre-eminent intellectual merit all over the country.

Central to the Akademi’s mission is the encouragement of a deeper understanding and enjoyment of
modern and contemporary art by a diverse local, national and international audience that it serves.
The National Exhibition of Art and the International Triennale – India, are examples of such
endeavors.

9 Consider the following statements regarding National Statistical Commission (NSC)


1. It was setup based on the recommendation of the Rangarajan Commission.
2. It serves as a nodal and empowered body for all core statistical activities of the country

prelims.insightsonindia.com 5
© Insights Active Learning | All rights reserved - 156493. You may not reproduce, distribute or exploit the contents in any form without
written permission by copyright owner. Copyright infringers may face civil and criminal liability
Total Marks : 200
Online Prelims TEST - 25 (SUBJECT)
( InsightsIAS Mock Test Series for UPSC Preliminary Exam 2020 ) Mark Scored : 31.33

3. The Chief Statistician of India is the Chairperson.

Which of the statements given above is/are correct?


A. 1 and 2 only
B. 2 and 3 only
C. 1 and 3 only
D. 1, 2 and 3

Your Answer : A
Correct Answer : A

Answer Justification :

National Statistical Commission

The National Statistical Commission (NSC) was set up in 2005. The setting up of the NSC
followed the decision of the Cabinet to accept the recommendation of the1Rangarajan 3
7
Commission, which reviewed the Indian Statistical System in 2001. The
2 52 NSC was initially
constituted in 2006, to serve as a nodal and empowered body for 8 all core statistical
6 3
- 8 priorities and standards and to
activities of the country, to evolve, monitor and enforce statistical
ensure statistical coordination. m
i l .co
m a
It has one part-time Chairperson and four part-time members, each having specialization and
g
experience in specified statistical fields. Besides, Secretary, Planning Commission is an ex-officio
0 5@
member of the Commission. The Chief Statistician of India is the Secretary to the
6
Commission. Hence statement 3 is
5 12incorrect.
m r7
o y-
10 Consider the following statements regarding National Council of Science Museums (NCSM)
1. It is an autonomous R
y body under the Ministry of Science and Technology
n
2. It is the largest j o
network of science centres/museums in the world functioning under a single
t u
i control.
Mr
administrative
3. NCSM is the implementing agency for the Science City Scheme.

Which of the statements given above is/are correct?


A. 1 and 2 only
B. 2 and 3 only
C. 1 and 3 only
D. 1, 2 and 3

Your Answer :
Correct Answer : B

Answer Justification :

National Council of Science Museums (NCSM), an autonomous body under the Ministry of
Culture, is the largest network of science centres/museums in the world functioning under
a single administrative control. Hence statement 1 is incorrect.

prelims.insightsonindia.com 6
© Insights Active Learning | All rights reserved - 156493. You may not reproduce, distribute or exploit the contents in any form without
written permission by copyright owner. Copyright infringers may face civil and criminal liability
Total Marks : 200
Online Prelims TEST - 25 (SUBJECT)
( InsightsIAS Mock Test Series for UPSC Preliminary Exam 2020 ) Mark Scored : 31.33

It is primarily engaged in spreading the culture of science in the society especially among students
with a motto of ‘Communicating Science to Empower People’ through its network of Science
Centres spread across India. NCSM is the implementing agency for the Science City Scheme
for developing new science centres in the country of national, regional and district level.

11 Annual Survey of Industries (ASI) is conducted by

A. National Sample Survey Office (NSSO)


B. Labour Bureau
C. Central Statistics Office
D. Reserve Bank of India (RBI)

Your Answer : A
Correct Answer : A

Answer Justification : 1 3
527
Annual Survey of Industries (ASI) is conducted by National Sample
3 82Survey Office (NSSO).
86event, not only facilitates
ASI is principal source of industrial statistics in India. ASI, an annual
-
suitable data collection based on appropriate sampling techniques but also ensures timely
dissemination of statistical information to asses and evaluate .c om
the dynamics in composition, growth
i l
and structure of organized manufacturing sector.Theastructure and function of the industrial sector
is an important perspective of Indian Economy. It
@ gmis imperative for industries to grow both
qualitatively and quantitatively to boost the0 5
economy.
2 6
7
The well-being of the industries depends 51
truly on the formulation and promotion of industrial
r
y -m
policies framed by the policy makers. To frame suitable industrial policies the policy makers need to

R o
be aware about the quantified aspect of the existing scenarios in the industries in the country. This

j oy Survey of Industries (ASI) is conducted by National Sample Survey


is where the Annual
n of India.
Office,Government
rit u
M
12 Consider the following statements regarding Members of Parliament Local Area Development
Scheme (MPLADS)
1. The scheme is implemented by Ministry of Parliamentary Affairs.
2. Under this scheme, Nominated Members of the Parliament can recommend works for
implementation, anywhere in the country.

Which of the statements given above is/are correct?


A. 1 only
B. 2 only
C. Both 1 and 2
D. Neither 1 nor 2

Your Answer : A
Correct Answer : B

Answer Justification :

prelims.insightsonindia.com 7
© Insights Active Learning | All rights reserved - 156493. You may not reproduce, distribute or exploit the contents in any form without
written permission by copyright owner. Copyright infringers may face civil and criminal liability
Total Marks : 200
Online Prelims TEST - 25 (SUBJECT)
( InsightsIAS Mock Test Series for UPSC Preliminary Exam 2020 ) Mark Scored : 31.33

The Members of Parliament Local Area Development Scheme (MPLADS) was launched in 1993.
Initially, Ministry of Rural Development was the Nodal Ministry for this scheme. In October, 1994
this scheme was transferred to the Ministry of Statistics and Programme Implementation.
Hence statement 1 is incorrect.

The objective of MPLAD Scheme is to enable MPs to recommend works of developmental


nature with emphasizes on creation of durable community assets in the areas of national
priorities, viz., drinking water facility; education; electricity facility; health and family welfare;
irrigation facility; non-conventional energy severs; railways, roads, pathways etc.

The salient features of the Scheme include:

a) The Scheme is fully funded by the Government of India under which funds are released in the
form of grants-in-aid directly to the district authorities.

b) The funds released under the Scheme are non-lapsable, i.e., the entitlement 7
3
of1funds not released
in a particular year is carried forward to the subsequent years, subject to 2 5 2
eligibility. At present, the
8
annual entitlement per MP/constituency is ₹ 5 crore. 63
-8
c) Under it, the role of the Members of Parliament is limited
.c om
to recommend works. Thereafter, it is

a il
the responsibility of the district authority to sanction, execute and complete the works
recommended within the stipulated time period. m
@ g
5
d) The elected Lok Sabha Members can
2 60 recommend works in their respective
51 of the Rajya Sabha can recommend works anywhere
constituencies. The elected members
7
in the state from which they rare elected. Nominated Members of the Parliament can
-m
recommend works for yimplementation, anywhere in the country.

y Ro
e) MPLADS works
n jo can be implemented in areas affected by natural calamities like floods, cyclone,
i u
hailstorm,tavalanche, cloudburst, pest attack, landslides, tornado, earthquake, drought, tsunami,
r
M biological, chemical, radiological hazards, etc.
fire and

f) In order to accord special attention to the development of areas inhabited by Scheduled Castes
(SCs) and Scheduled Tribes (STs), 15 per cent of MPLADS funds are to be utilized for areas
inhabited by SC population and 7.5 per cent for areas inhabited by ST population.

g) If an elected Member of Parliament finds the need, to contribute these funds, to a place outside
that state/UT or outside the constituency within the state or both, the MP can recommend eligible
works, under these Guidelines upto a maximum of ₹ 25 lakh in a financial year. Such a gesture will
promote national unity, harmony, and fraternity among the people, at the grass roots level.

h) The MP can spend a maximum of ₹ 20 lakh per year for giving assistance to Differently Abled
Citizens for purchase of tri-cycles (including motorized tricycles), battery operated motorized wheel
chair and artificial limbs; and aids for visually and hearing impaired.

13 Consider the following statements regarding Special Economic Zones (SEZs) Policy
1. Asia’s first Export Processing Zone (EPZ) was set up in Kandla in 1965.

prelims.insightsonindia.com 8
© Insights Active Learning | All rights reserved - 156493. You may not reproduce, distribute or exploit the contents in any form without
written permission by copyright owner. Copyright infringers may face civil and criminal liability
Total Marks : 200
Online Prelims TEST - 25 (SUBJECT)
( InsightsIAS Mock Test Series for UPSC Preliminary Exam 2020 ) Mark Scored : 31.33

2. It intends to make SEZs an engine for economic growth supported by quality infrastructure with
attractive fiscal package and minimum possible regulations.

Which of the statements given above is/are correct?


A. 1 only
B. 2 only
C. Both 1 and 2
D. Neither 1 nor 2

Your Answer :
Correct Answer : C

Answer Justification :

India was one of the first in Asia to recognise the effectiveness of the Export Processing Zone (EPZ)
model in promoting exports, with Asia’s first EPZ set up in Kandla in 1965. To overcome the
13
shortcomings on account of multiplicity of controls and clearances, absence of world-class
7
infrastructure and an unstable fiscal regime and with a view to attract larger
2 52foreign investments in
India, the Special Economic Zones (SEZs) Policy was announced3in 8 April 2000. This policy
6
intended to make SEZs an engine for economic growth supported
- 8 by quality infrastructure
complemented by an attractive fiscal package, both at the
.c om Centre and the state level, with
minimum possible regulations. il
gma
5 System
14 Consider the following statements regarding0Postal
@
6
12 in India by Lord Clive.
1. The modern postal system was established
5
r7 engines of social development.
2. Post office became one of the greatest
m
- the postal services in the country is the Indian Post Office Act,
3. The statute presently governing
y
1898. o
j oyR
Which of the
i t un
statements given above is/are correct?
A. 1 r
Mand 2 only
B. 2 and 3 only
C. 1 and 3 only
D. 1, 2 and 3

Your Answer :
Correct Answer : D

Answer Justification :

The modern postal system, the most preferred facilitator of communication, was
established in India by Lord Clive in 1766 and it was further developed by Warren Hastings in
1774. The expansion of its network was made during 1786 to 1793. For the first time, the post
offices were regulated through an Act of 1837 on a uniform basis to unite all the post office
establishments throughout the then existing three Presidencies into one all-India service.
Thereafter, the Post Office Act of 1854 reformed the entire fabric of the postal system and the post
offices of India were placed on the present administrative footing on 1st October 1854.

prelims.insightsonindia.com 9
© Insights Active Learning | All rights reserved - 156493. You may not reproduce, distribute or exploit the contents in any form without
written permission by copyright owner. Copyright infringers may face civil and criminal liability
Total Marks : 200
Online Prelims TEST - 25 (SUBJECT)
( InsightsIAS Mock Test Series for UPSC Preliminary Exam 2020 ) Mark Scored : 31.33

Even though the British established the post office for imperial interests, it along with the
railways and telegraph, became one of the greatest engines of social development.

The statute presently governing the postal services in the country is the Indian Post Office
Act, 1898. In the mid-19th century, the post office served as facilitator of travel with its
conveyance systems and by maintaining the dak bungalows and dak serais. Mail order services
were started with the value payable system introduced in 1877, while fund remittances at the
doorstep became possible from 1880 through money order services. With the introduction of the
Post Office Savings Bank in 1882 banking facilities were accessible to all and by 1884 all
government employees were covered by the postal life insurance. Besides providing postal
communication facilities, the post office network has also provided facilities for remittance of funds,
banking and insurance services since the latter half of the 19th century.

15 Consider the following statements regarding Tele-density


1. It denotes the number of telephones per 100 population.
3
2. The rural tele-density is 56.58 per cent while that in urban areas it is 67.50 per cent.
1
3. Tamil Nadu has the highest tele-density in India. 2 7
2 5
6 38
-8
Which of the statements given above is/are correct?
A. 1 only
B. 2 and 3 only .c om
C. 1 and 3 only a il
m
D. 1, 2 and 3 @g 5
0
Your Answer : B 126
5
Correct Answer : A r7
y -m
Answer Justification o
y R:
jo denotes the number of telephones per 100 population is an important
nwhich
Tele-density, u
ioft telecom penetration in any country. Tele-density in India-was 91.64 per cent at the end
r
indicator
M
of November 2017. The rural tele-density is now 56.58 per cent while that in urban areas it
is 167.50 per cent. Hence statement 2 is incorrect.

Amongst the service areas, Himachal Pradesh (153.96 per cent) had the highest tele-
density followed by Tamil Nadu (124.38 per cent), Punjab (123.62 per cent), Kerala (118.58 per
cent) and Gujarat (110,00 per cent). Hence statement 3 is incorrect. On the other hand, it is
comparatively low in service areas such as Bihar (60.13 per cent), Assam (68.41 per cent), Madhya
Pradesh (69,47 per cent), Uttar Pradesh (69.66 per cent), West Bengal (72.90 per cent) and Odisha
(79.58 per cent), Amongst the metros, Delhi tops in tele-density with 259.14 per cent, followed by
Kolkata

(184.56 per cent) and Mumbai (169.97 per cent).

16 Consider the following statements regarding Mahanagar Telephone Nigam Limited (MTNL)
1. It is a Navratna PSU and provides telecommunication facilities in all metro cities in India.
2. MTNL is providing triple play services i.e., voice, high speed internet and IPTV on its broadband

prelims.insightsonindia.com 10
© Insights Active Learning | All rights reserved - 156493. You may not reproduce, distribute or exploit the contents in any form without
written permission by copyright owner. Copyright infringers may face civil and criminal liability
Total Marks : 200
Online Prelims TEST - 25 (SUBJECT)
( InsightsIAS Mock Test Series for UPSC Preliminary Exam 2020 ) Mark Scored : 31.33

network.

Which of the statements given above is/are correct?


A. 1 only
B. 2 only
C. Both 1 and 2
D. Neither 1 nor 2

Your Answer : B
Correct Answer : B

Answer Justification :

Mahanagar Telephone Nigam Limited (MTNL), set up in 1986, is a Navratna PSU and
provides telecommunication facilities in India’s key metros - Delhi and Mumbai. Hence
statement 1 is incorrect. MTNL is the principal provider of fixed-line telecommunication service
13 of Noida,
in these two metropolitan cities, and for GSM mobile services in four peripheral towns
7
Gurgaon, Faridabad and Ghaziabad along with Delhi city and the areas falling
2 52under the Mumbai
Municipal Corporation, New Mumbai Corporation and Thane Municipal
6 38Corporation along with
- 8 is providing triple play
Mumbai city, also come under the jurisdiction of the company. MTNL
services i.e., voice, high speed internet and IPTV on itsm broadband network.
.c o
m and ail
17 Al Nagah is a defence exercise conducted betweengIndia
5 @
2 60
A. UAE 1
B. Iran r 75
m
y-
C. Saudi Arabia
o
yR
D. Oman

n j o
it u
Your Answer :
Correct rAnswer : D
M
Answer Justification :

Al Nagah

Second edition of ‘Al Nagah’ the joint exercise between the Indian and Oman Armies was
held at Bakloh, Himachal Pradesh. Both armies shared valuable combat expertise on countering
terrorist operations in an international environment.

18 Consider the following statements regarding Shaala Siddhi


1. It is developed by the National University of Educational Planning and Administration (NUEPA).
2. It is a comprehensive instrument for evaluation of student’s performance and focus on
improvements in weak areas.

Which of the statements given above is/are correct?


A. 1 only

prelims.insightsonindia.com 11
© Insights Active Learning | All rights reserved - 156493. You may not reproduce, distribute or exploit the contents in any form without
written permission by copyright owner. Copyright infringers may face civil and criminal liability
Total Marks : 200
Online Prelims TEST - 25 (SUBJECT)
( InsightsIAS Mock Test Series for UPSC Preliminary Exam 2020 ) Mark Scored : 31.33

B. 2 only
C. Both 1 and 2
D. Neither 1 nor 2

Your Answer :
Correct Answer : A

Answer Justification :

Shaala Siddhi: School Standards and Evaluation Framework and its web portal was launched in
2015. It is a comprehensive instrument for school evaluation leading to school
improvement. Hence statement 2 is incorrect.

Developed by the National University of Educational Planning and Administration


(NUEPA), it aims to enable schools to evaluate their performance in a more focused and strategic
manner and facilitate them to make professional judgements for improvement. The programme’s
13 by focussing on
objective is to establish an agreed set of standards and benchmarks for each school,
7
key performance domains and their core standards. 2 5
2
38
19 Consider the following statements regarding schemes in Education- sector
86
c om Institutions.
1. Prashikshak portal is helpful in monitoring of teacher’s education
.
2. SAKSHAM is a Scholarship Scheme for differently-abledail children
@ gm
5
0correct?
Which of the statements given above is/are
6
A. 1 only
5 12
B. 2 only
m r7
C. Both 1 and 2 -
D. Neither 1 nor 2 oy

j oyR
n
it u : C
Your Answer :
Correct rAnswer
M
Answer Justification :

Both the statements are correct.

India Teacher Education Portal (Prashikshak): The Department of School Education and Literacy,
launched ‘India Teacher Education Portal (Prashikshak)’ in 2016. This portal is helpful in
monitoring of teacher’s education Institutions and in providing comprehensive information to
prospective students and teachers to select the right institute as per choice.

SAKSHAM - a Scholarship for Differently-Abled Children

SAKSHAM - Scholarship Scheme for differently-abled children aims at providing


encouragement and support to differently-abled children to pursue technical education.
Scholarships amounting to ₹ 5 crore per annum as tuition fees and incidentals are to be provided to
needy and meritorious students for pursuing technical education at AICTE approved institution.
This is to help them to achieve their college goals, despite learning issues, environmental

prelims.insightsonindia.com 12
© Insights Active Learning | All rights reserved - 156493. You may not reproduce, distribute or exploit the contents in any form without
written permission by copyright owner. Copyright infringers may face civil and criminal liability
Total Marks : 200
Online Prelims TEST - 25 (SUBJECT)
( InsightsIAS Mock Test Series for UPSC Preliminary Exam 2020 ) Mark Scored : 31.33

challenges or medical bills.

20 Consider the following statements regarding Copyright


1. Acquisition of copyright is automatic and it does not require any formality.
2. The Copyright Office was established under the administrative control of the Department for
Promotion of Industry and Internal Trade.

Which of the statements given above is/are correct?


A. 1 only
B. 2 only
C. Both 1 and 2
D. Neither 1 nor 2

Your Answer : B
Correct Answer : A
1 3
Answer Justification : 5 27
2
6 38
-8
Copyright

o m
Acquisition of copyright is automatic and it does not
i l .crequire any formality. Copyright comes
a
into existence as soon as a work is created and no formality
mcertificate
is required to be completed for

@ g
acquiring it. However, as per Section 48 of the Act, of registration of copyright and the
entries made therein serve as prima facie 0 5
evidence in a court of law with reference to dispute
relating to ownership of copyright. 12 6
r 75
- m
The Copyright Office was established in 1958. It functions under the administrative
oy of Higher Education. Hence statement 2 is incorrect.
control of the Department
R
j oy
It is headed by
u n Registrar of Copyright, who has quasi-judicial powers in handling cases relating
the
r t
i The main function of the Copyright Office is to undertake registration of copyright.
to copyright.
M
The Register of Copyrights provides information regarding works of copyright to the general public.
In addition to registration, facilities like inspection of the Register and taking extracts thereof are
also available in the Copyright Office.

21 Consider the following statements regarding Coal India Limited (CIL)


1. It is a ‘Maha Ratna’ company under the Ministry of Coal.
2. It is the single largest coal producing company in the world.

Which of the statements given above is/are correct?


A. 1 only
B. 2 only
C. Both 1 and 2
D. Neither 1 nor 2

Your Answer :
Correct Answer : C

prelims.insightsonindia.com 13
© Insights Active Learning | All rights reserved - 156493. You may not reproduce, distribute or exploit the contents in any form without
written permission by copyright owner. Copyright infringers may face civil and criminal liability
Total Marks : 200
Online Prelims TEST - 25 (SUBJECT)
( InsightsIAS Mock Test Series for UPSC Preliminary Exam 2020 ) Mark Scored : 31.33

Answer Justification :

Coal India Limited (CIL) is a ‘Maha Ratna’ company under the Ministry of Coal, with
headquarters at Kolkata, West Bengal. CIL is the single largest coal producing company in
the world and one of the largest corporate employers with a manpower of 3,46,638. CIL operates
through 82 mining areas spread over eight provincial states of India. It has 429 mines of which 237
are underground, 166 opencast and 26 mixed mines.

CIL is the apex body in coal industry under the administrative control of the Ministry of
Coal. Coal India is a holding company with seven wholly owned coal producing subsidiary
companies and one mine planning and consultancy company. It encompasses the whole gamut of
identification of coal reserves, detailed exploration followed by design and implementation and
optimising operations for coal extraction in its mines.

22 Consider the following statements regarding Polypedates Bengalensis


1. It is a newly discovered frog species from West Bengal.
1 3
2. There more than 250 other Polypedates species round the world
5 27
2
Which of the statements given above is/are correct? 6 38
A. 1 only -8
B. 2 only
.c om
C. Both 1 and 2 a il
D. Neither 1 nor 2 gm @
5
2 60
Your Answer :
1
Correct Answer : A
r 75
m
Answer Justification o
: y-
j oyR
u n
Polypedates Bengalensis is a newly discovered frog species from West Bengal. Also known as the
ri t
Brown Blotched Bengal Tree Frog It belongs to the genus Polypedates.. Hence Statement 1 is
M
correct

There are 25 other Polypedates species round the world. Polypedates bengalensis is the 26th. The
frog’s body colour is yellowish-brown to greenish-brown. Hence Statement 2 is incorrect

23 Consider the following statements regarding National Broadband Mission


1. It is launched by Ministry of Communications
2. The mission aimed at providing broadband access in all villages in the country by 2030.
3. Under the mission, the government plans to lay incremental 30 lakh route km of Optical Fiber
Cable.

Which of the statements given above is/are correct?


A. 1 and 3 only
B. 2 only
C. 2 and 3 only
D. 1, 2 and 3

prelims.insightsonindia.com 14
© Insights Active Learning | All rights reserved - 156493. You may not reproduce, distribute or exploit the contents in any form without
written permission by copyright owner. Copyright infringers may face civil and criminal liability
Total Marks : 200
Online Prelims TEST - 25 (SUBJECT)
( InsightsIAS Mock Test Series for UPSC Preliminary Exam 2020 ) Mark Scored : 31.33

Your Answer : B
Correct Answer : A

Answer Justification :

The Ministry of Communications has launched ‘National Broadband Mission’ that will facilitate
universal and equitable access to broadband services across the country, especially in rural and
remote areas.17 . Hence Statement 1 is correct

The mission aimed at providing broadband access in all villages in the country by 2022. . Hence
Statement 2 is incorrect

Under the mission, the government plans to lay incremental 30 lakh route km of Optical Fiber
Cable. The Centre will work with States and UTs for having consistent policies pertaining to
expansion of digital infrastructure including for Right of Way (RoW) approvals required for laying of
optical fibre cable. . Hence Statement 3 is correct.
1 3
5 27
24 Consider the following statements regarding Nirvik Scheme 2
1. It is introduced by Ministry of Finance 6 38
2. The scheme is valid for 10 years. -8
om
3. It is to enhance loan availability and ease the lending process
.c
a il
gm
Which of the statements given above is/are correct?
@
A. 1 and 3 only
6 05
12
B. 1, 2 and 3
5
C. 3 only
D. 1 and 3 only m r7
o y-
Your Answer : C y R
Correct Answer
u njo: C
r it
M
Answer Justification :

Nirvik Scheme is a new Export Credit Insurance Scheme (ECIS) introduced by Ministry of
Commerce and Industry through Export Credit Guarantee Corporation (ECGC). Hence Statement
1 is incorrect.

Under the scheme, ECGC will provide 90% credit insurance cover and any additional outgo would
be supported by the government. The scheme is valid for 5 years. Hence Statement 2 is
incorrect.

It is to enhance loan availability and ease the lending process. It would give a fillip to export lending
and insurance cover for export credit. Hence Statement 3 is correct.

25 Consider the following statements regarding Hunar Haat


1. Hunar Haat are organized by Ministry of Culture
2. It is aimed at promoting and supporting artisans from Minority and tribal communities

prelims.insightsonindia.com 15
© Insights Active Learning | All rights reserved - 156493. You may not reproduce, distribute or exploit the contents in any form without
written permission by copyright owner. Copyright infringers may face civil and criminal liability
Total Marks : 200
Online Prelims TEST - 25 (SUBJECT)
( InsightsIAS Mock Test Series for UPSC Preliminary Exam 2020 ) Mark Scored : 31.33

Which of the statements given above is/are correct?


A. 1 only
B. 2 only
C. Both 1 and 2
D. Neither 1 nor 2

Your Answer : C
Correct Answer : D

Answer Justification :

Hunar Haats are organised by Ministry of Minority Affairs under USTTAD (Upgrading the Skills &
Training in Traditional Arts/Crafts for Development) scheme. Hence Statement 1 is incorrect

Hunar Haat is an exhibition of handicrafts and traditional products made by artisans from the
Minority communities. Aimed at promoting and supporting artisans from Minority communities and
13their products.
providing them domestic as well as international markets for displaying and selling
7
Hence Statement 2 is incorrect. 52 2
8
8 63
26 Consider the following statements regarding Nari Shakti Puraskar-
1. The Awards were initiated in the year 2015. .c om
a il these national level awards for eminent
2. The Ministry of Women and Child Development announces
women, organisations and institutions.
@ gm
05 of Rs.1 Lakh.
3. The Nari Shakti Puraskar carries a cash award
6
2
51
7 is/are correct?
r
Which of the statements given above
A. 2 and 3 only
y -m
B. 3 only
C. 1, 2 and 3 oy
Ro
D. None un
j
rit
M
Your Answer :
Correct Answer : A

Answer Justification :

To acknowledge Women’s achievements, the Government of India confers Nari Shakti Puraskars on
eminent women and institutions in recognition of their service towards the cause of women
empowerment. The Awards were initiated in the year 1999. Hence Statement 1 is incorrect

The Ministry of Women and Child Development announces these national level awards for eminent
women, organisations and institutions. Hence Statement 2 is correct

The Nari Shakti Puraskar carries a cash award of Rs.1 Lakh and a certificate for individuals and
institutions. Hence Statement 3 is correct

27 Consider the following statements regarding SAANS Initiative

prelims.insightsonindia.com 16
© Insights Active Learning | All rights reserved - 156493. You may not reproduce, distribute or exploit the contents in any form without
written permission by copyright owner. Copyright infringers may face civil and criminal liability
Total Marks : 200
Online Prelims TEST - 25 (SUBJECT)
( InsightsIAS Mock Test Series for UPSC Preliminary Exam 2020 ) Mark Scored : 31.33

1. It is an initiative under National Health Mission.


2. It is to accelerate action to reduce deaths due to Childhood Pneumonia.
3. Under this, the government is targeting a reduction in pneumonia-caused deaths by 2025 to less
than 3 deaths out of 1000 live births.

Which of the statements given above is/are correct?


A. 1 and 3 only
B. 2 only
C. 2 and 3 only
D. 1, 2 and 3

Your Answer : D
Correct Answer : D

Answer Justification :

13initiative under
Social Awareness and Actions to Neutralize Pneumonia Successfully (SAANS) is an
7
National Health Mission. Hence Statement 1 is correct 52
3 82
86 Hence Statement 2 is
It is to accelerate action to reduce deaths due to Childhood Pneumonia.
-
om
correct
l .c
ipneumonia-caused
Under this, the government is targeting a reduction in
m a deaths by 2025 to less
g
than 3 deaths out of 1000 live births. Hence Statement 3 is correct
0 5@
Under SAANS, health and wellness centres
1 26 can use pulse oximeter (device to monitor oxygen
5 in the blood of a child, and if required, treat him by use of
saturation) to identify low oxygen7levels
r
oxygen cylinders.
y -m
y Ro
njo statements regarding Chief of Defence Staff (CDS)
28 Consider the following
u
1. CDS actsias
r t the Secretary of Department of Military Affairs.
2. CDS M promotes the use of indigenous equipment by the services

Which of the statements given above is/are correct?


A. 1 only
B. 2 only
C. Both 1 and 2
D. Neither 1 nor 2

Your Answer : B
Correct Answer : C

Answer Justification :

Both the statements given above are correct.

Chief of Defence Staff (CDS) acts as the Secretary of Department of Military Affairs.

prelims.insightsonindia.com 17
© Insights Active Learning | All rights reserved - 156493. You may not reproduce, distribute or exploit the contents in any form without
written permission by copyright owner. Copyright infringers may face civil and criminal liability
Total Marks : 200
Online Prelims TEST - 25 (SUBJECT)
( InsightsIAS Mock Test Series for UPSC Preliminary Exam 2020 ) Mark Scored : 31.33

As per the Second Schedule to Government of India (Allocation of Business) Rules 1961, the
following subjects were allocated to DMA:-

The Armed Forces of the Union, namely, Army, Navy and Air Force.

Integrated Headquarters of the Ministry of Defence comprising of Army Headquarters,


Naval Headquarters, Air Headquarters and Defence Staff Headquarters.

The Territorial Army.

Works relating to Army, Navy and Air Force.

3 prevalent rules
Procurement exclusive to the Services except capital acquisitions, as per
1
and procedures. 27 5
2
6 38
-8
Promoting use of indigenous equipment by the Services.
o m
il.c
ma Civilian officers and staff have been
Work of 23 sections along with around 160
g
@ to DMA.
transferred from Department of Defence
5
0
1 26
75
https://pib.gov.in/PressReleseDetailm.aspx?PRID=1601813
r
m
o y-
29 Consider the followingR
j oy statements regarding Classical Swine Fever (CSF)
unsurvive in pork and processed pork products for months when meat is refrigerated
1. It is a contagious viral disease of domestic and wild swine.
2. CSF virusitcan
Mryears when it is frozen.
and for
3. Humans are affected by this virus.

Which of the statements given above is/are correct?


A. 3 only
B. 2 and 3 only
C. 1 and 2 only
D. 1, 2 and 3

Your Answer : C
Correct Answer : C

Answer Justification :

Classical swine fever (CSF), also known as hog cholera, is a contagious viral disease of domestic
and wild swine. It is caused by a virus of the genus Pestivirus of the family Flaviviridae, which is
closely related to the viruses that cause bovine viral diarrhoea in cattle and border disease in sheep.

prelims.insightsonindia.com 18
© Insights Active Learning | All rights reserved - 156493. You may not reproduce, distribute or exploit the contents in any form without
written permission by copyright owner. Copyright infringers may face civil and criminal liability
Total Marks : 200
Online Prelims TEST - 25 (SUBJECT)
( InsightsIAS Mock Test Series for UPSC Preliminary Exam 2020 ) Mark Scored : 31.33

Transmission and spread

The most common method of transmission is through direct contact between healthy swine and
those infected with CSF virus. The virus is shed in saliva, nasal secretions, urine, and feces. Contact
with contaminated vehicles, pens, feed, or clothing may spread the disease. Animals that are
chronic carriers of the disease (persistently infected) may show no clinical signs of illness but may
shed the virus in their feces. Offspring of infected sows can become infected in the uterus, and can
shed the virus for months.

CSF virus can survive in pork and processed pork products for months when meat is refrigerated
and for years when it is frozen. Pigs can become infected by eating CSF-infected pork meat or
products.

It has been proven that in parts of Europe, the wild boar population may play a role in the
epidemiology of the disease.

The disease has been spread through legal and illegal transport of animals, and by3feeding swill
containing infective tissues to pigs. 7 1
5 2
3 82
Public health risk 6
-8
om known to be susceptible.
Humans are not affected by this virus. Swine are the only species

a il.c
gm
https://www.oie.int/en/animal-health-in-the-world/animal-diseases/Classical-swine-fever/
@
05
https://pib.gov.in/PressReleseDetailm.aspx?PRID=1601778
6
5 12
30 Tishreen Revolution, sometimem
r7
seen in the news, is witnessed in which country?
o y-
A. Iran
j oyR
B. Iraq un
rit
C. Syria
M
D. Venezuela

Your Answer : C
Correct Answer : B

Answer Justification :

The 2019 Iraqi protests have been named Tishreen Revolution or October Revolution or Iraqi
Intifada. It was started in October 2019 in the social media by civil activists.

https://www.thehindu.com/news/international/iraqi-cleric-asks-followers-to-stop-stir-clear-the-roads/
article30721682.ece

31 Surajkund International crafts, sometime seen in the news, is held in

A. Rajasthan

prelims.insightsonindia.com 19
© Insights Active Learning | All rights reserved - 156493. You may not reproduce, distribute or exploit the contents in any form without
written permission by copyright owner. Copyright infringers may face civil and criminal liability
Total Marks : 200
Online Prelims TEST - 25 (SUBJECT)
( InsightsIAS Mock Test Series for UPSC Preliminary Exam 2020 ) Mark Scored : 31.33

B. Haryana
C. Punjab
D. None of the above

Your Answer :
Correct Answer : B

Answer Justification :

http://www.haryanatourism.gov.in/Events/surajkund-international-crafts-mela-faridabad

32 Consider the following statements regarding Union Budget 2020-21


1. Estimated revenue receipts for 2020-21 is higher than capital receipts
2. Estimated Primary deficit for 2020-21 is lesser than 2019-20 budget estimates

Which of the statements given above is/are correct?


1 3
A. 1 only
5 27
2
B. 2 only
6 38
-8
C. Both 1 and 2
D. Neither 1 nor 2
o m
il.c
Your Answer :
Correct Answer : A gma
5 @
0
Answer Justification : 126
5
m r7
-
oy
j oyR
n
rit u
M

33 Consider the following statements regarding Union Budget


1. Corporate tax contributes to 25% of income of the government.
prelims.insightsonindia.com 20
© Insights Active Learning | All rights reserved - 156493. You may not reproduce, distribute or exploit the contents in any form without
written permission by copyright owner. Copyright infringers may face civil and criminal liability
Total Marks : 200
Online Prelims TEST - 25 (SUBJECT)
( InsightsIAS Mock Test Series for UPSC Preliminary Exam 2020 ) Mark Scored : 31.33

2. State share of duties and taxes forms the 25% of government expenditure.

Which of the statements given above is/are correct?


A. 1 only
B. 2 only
C. Both 1 and 2
D. Neither 1 nor 2

Your Answer :
Correct Answer : D

Answer Justification :

1 3
5 27
2
6 38
-8
o m
il.c
gma
5 @
0
126
5
m r7
-
oy
j oyR
n
rit u
M

34 Consider the following statements regarding National Mission on Quantum Technologies &
Applications
1. The mission was announced for a period of five years to be implemented by the Department of
Science & Technology (DST).
2. Quantum principles will be used for engineering solutions to extremely complex problems
in computing, communications, sensing, chemistry, cryptography, imaging and mechanics.

Which of the statements given above is/are correct?


A. 1 only
B. 2 only
C. Both 1 and 2
D. Neither 1 nor 2

prelims.insightsonindia.com 21
© Insights Active Learning | All rights reserved - 156493. You may not reproduce, distribute or exploit the contents in any form without
written permission by copyright owner. Copyright infringers may face civil and criminal liability
Total Marks : 200
Online Prelims TEST - 25 (SUBJECT)
( InsightsIAS Mock Test Series for UPSC Preliminary Exam 2020 ) Mark Scored : 31.33

Your Answer : B
Correct Answer : C

Answer Justification :

Both the statements are correct.

National Mission on Quantum Technologies & Applications

The mission was announced for a period of five years to be implemented by the Department of
Science & Technology (DST). Quantum technology is opening up new frontiers in computing,
communications, cyber security with wide-spread applications.

Quantum principles will be used for engineering solutions to extremely complex problems
in computing, communications, sensing, chemistry, cryptography, imaging and mechanics.

https://pib.gov.in/PressReleseDetailm.aspx?PRID=1601563
1 3
527
35 Consider the following statements regarding Yellow Rust Disease
3 82
6
- 8colour.
1. It is a fungal disease which turns the crop’s leaves into a yellowish
m
2. It is a disease of cool weather in the northern hills and north-western plains zone.
o
.c
ail
gm
Which of the statements given above is/are correct?
A. 1 only @
B. 2 only 6 05
C. Both 1 and 2 5 12
D. Neither 1 nor 2 r7 m
-
Your Answer : C R oy
Correct Answerjo
y
:C
i t un
MrJustification :
Answer

Both the statements are correct.

Yellow rust is a fungal disease which turns the crop’s leaves into a yellowish colour. It stops
photosynthesis activity, which eventually could result in a drop of wheat crop productivity.

https://indianexpress.com/article/explained/this-word-means-yellow-rust-6245018/

36 Consider the following statements regarding Indian National Commission for Cooperation with
UNESCO
1. It was set up in 1949 by the Government of India.
2. It functions under the Ministry of Culture.
3. The main objective of the Commission is to advise the Government in matters falling in the domain

prelims.insightsonindia.com 22
© Insights Active Learning | All rights reserved - 156493. You may not reproduce, distribute or exploit the contents in any form without
written permission by copyright owner. Copyright infringers may face civil and criminal liability
Total Marks : 200
Online Prelims TEST - 25 (SUBJECT)
( InsightsIAS Mock Test Series for UPSC Preliminary Exam 2020 ) Mark Scored : 31.33

of UNESCO.

Which of the statements given above is/are correct?


A. 1 only
B. 3 only
C. 1 and 3 only
D. 1, 2 and 3

Your Answer : D
Correct Answer : C

Answer Justification :

Indian National Commission for Cooperation with UNESCO

It was set up in 1949 by the Government of India. A permanent Commission was established in
1951. Hence Statement 1 is correct. 13
5 27
It functions under the Department of Secondary and Higher Education in
3 82the Ministry of Human
6
-8
Resource Development. Hence Statement 2 is incorrect.

c om in matters falling in the domain


The main objective of the Commission is to advise the Government
.
a il
of UNESCO and to play a role in UNESCO’s work particularly in the formulation and execution of its
programmes. Hence Statement 3 is correct. gm
0 5@
1 26
The Commission consists of five Sub-Commissions namely:
5
Education m r7
Natural Sciences y
-
o
j oyR
Social Sciences

un
Communication
i t
Mr
Culture.

Composition:

President: The Minister of Human Resource Development.

Secretary General: The Secretary in the Department of Higher Education, Ministry of Human
Resource Development.

https://pib.gov.in/newsite/PrintRelease.aspx?relid=197753

37 Consider the following statements regarding Neglected Tropical Diseases (NTDs)


1. They are a diverse group of communicable diseases that prevail in tropical and subtropical
conditions in 149 countries.
2. According to the WHO, 30 health conditions are categorized as Neglected tropical diseases.

prelims.insightsonindia.com 23
© Insights Active Learning | All rights reserved - 156493. You may not reproduce, distribute or exploit the contents in any form without
written permission by copyright owner. Copyright infringers may face civil and criminal liability
Total Marks : 200
Online Prelims TEST - 25 (SUBJECT)
( InsightsIAS Mock Test Series for UPSC Preliminary Exam 2020 ) Mark Scored : 31.33

3. Dengue and Chikungunya are the Neglected Tropical Diseases.

Which of the statements given above is/are correct?


A. 1 and 2 only
B. 2 and 3 only
C. 1 and 3 only
D. 1, 2 and 3

Your Answer : C
Correct Answer : C

Answer Justification :

Neglected Tropical Diseases (NTDs) are a diverse group of communicable diseases that prevail in
tropical and subtropical conditions in 149 countries. Hence Statement 1 is correct.

3
They are caused by a variety of pathogens such as viruses, bacteria, protozoa and1helminthes.
5 27
82 health conversation.
NTDs are considered diseases of poverty and are often left out of the global
3
86
Populations living in poverty, without adequate sanitation and in-close contact with infectious
m
oaffected.
vectors and domestic animals and livestock are those worst
il .c
a
@ gm
They blind, disfigure, and debilitate people in the poorest regions of the world.

According to the WHO, 20 health conditions6 05 are categorized as Neglected tropical diseases.
2
Hence Statement 2 is incorrect.51
m r7
Buruli ulcer
o y-
Chagas diseaseR
Dengue andj oy Chikungunya. Hence Statement 3 is correct.
i t un
Dracunculiasis (guinea-worm disease)
M r
Echinococcosis
Foodborne trematodiases
Human African trypanosomiasis (sleeping sickness)
Leishmaniasis
Leprosy (Hansen’s disease)
Lymphatic filariasis
Mycetoma, chromoblastomycosis and other deep mycoses
Onchocerciasis (river blindness)
Rabies
Scabies and other ectoparasites
Schistosomiasis
Soil-transmitted helminthiases
Snakebite envenoming
Taeniasis/Cysticercosis
Trachoma
Yaws (Endemic treponematoses)

prelims.insightsonindia.com 24
© Insights Active Learning | All rights reserved - 156493. You may not reproduce, distribute or exploit the contents in any form without
written permission by copyright owner. Copyright infringers may face civil and criminal liability
Total Marks : 200
Online Prelims TEST - 25 (SUBJECT)
( InsightsIAS Mock Test Series for UPSC Preliminary Exam 2020 ) Mark Scored : 31.33

https://indianexpress.com/article/india/what-are-neglected-tropical-diseases-that-affect-more-than-a-
billion-people-6243130/

38 Consider the following statements regarding Classical Languages


1. Sanskrit was the first language to be declared as Classical language.
2. Two major annual international awards for scholars of eminence in classical Indian languages.

Which of the statements given above is/are correct?


A. 1 only
B. 2 only
C. Both 1 and 2
D. Neither 1 nor 2

Your Answer :
Correct Answer : B
1 3
Answer Justification : 527
2
38
86 of India, awarded by the
Classical Language in India is an official status within the Republic
-
Government of India.
om
a il.c
gm status:
There are six languages enjoy the ‘Classical’
@
5
2 60
51Hence Statement 1 is incorrect.
Tamil (declared in 2004).
7
r
y -m
o
Sanskrit (2005)
j oyR
n
rituKannada (2008)
M
Telugu (2008)

Malayalam (2013)

Odia (2014)

Criteria’s for declaring a language as ‘Classical’ are:

High antiquity of its early texts/recorded history over a period of 1500-2000 years.

A body of ancient literature/texts, which is considered a valuable heritage by generations of

prelims.insightsonindia.com 25
© Insights Active Learning | All rights reserved - 156493. You may not reproduce, distribute or exploit the contents in any form without
written permission by copyright owner. Copyright infringers may face civil and criminal liability
Total Marks : 200
Online Prelims TEST - 25 (SUBJECT)
( InsightsIAS Mock Test Series for UPSC Preliminary Exam 2020 ) Mark Scored : 31.33

speakers.

The literary tradition be original and not borrowed from another speech community.

The classical language and literature being distinct from modern, there may also be a
discontinuity between the classical language and its later forms or its offshoots.

Benefits Once A Language Is Notified As A Classical Language:

Two major annual international awards for scholars of eminence in classical Indian
languages. Hence Statement 2 is correct.

A Centre of Excellence for studies in Classical Languages is set up


1 3
27
25
38with at least in the Central
The University Grants Commission is requested to create, to start
6
8 Classical Languages so
Universities, a certain number of Professional Chairs for-the
declared. om
a il.c
@ gm
https://indianexpress.com/article/explained/explained-how-is-a-language-declared-classical-in-india-
what-benefits-it-enjoys-6216415/ 6 05
5 12
m r7
o y-
j oy Rstatements regarding Commission for Agricultural Costs and Prices (CACP)
39 Consider the following
1. CACP submitsnits recommendations to the government in the form of Price Policy Reports every five
year. rit
u
M
2. Commission comprises a Chairman, Member Secretary, one Member (Official) and two Members
(Non-Official).
3. It is mandated to recommend minimum support prices (MSPs) to incentivize the cultivators to adopt
modern technology, and raise productivity and overall grain production in line with the emerging
demand patterns in the country.

Which of the statements given above is/are correct?


A. 2 and 3 only
B. 3 only
C. 1 and 3 only
D. 1, 2 and 3

Your Answer :
Correct Answer : A

Answer Justification :

prelims.insightsonindia.com 26
© Insights Active Learning | All rights reserved - 156493. You may not reproduce, distribute or exploit the contents in any form without
written permission by copyright owner. Copyright infringers may face civil and criminal liability
Total Marks : 200
Online Prelims TEST - 25 (SUBJECT)
( InsightsIAS Mock Test Series for UPSC Preliminary Exam 2020 ) Mark Scored : 31.33

About Commission for Agricultural Costs and Prices (CACP)

The Commission for Agricultural Costs & Prices (CACP) is an attached office of the Ministry of
Agriculture and Farmers Welfare, Government of India. It came into existence in January 1965.

It is mandated to recommend minimum support prices (MSPs) to incentivize the cultivators to adopt
modern technology, and raise productivity and overall grain production in line with the emerging
demand patterns in the country. Hence Statement 3 is correct.

Composition

Currently, the Commission comprises a Chairman, Member Secretary, one Member


(Official) and two Members (Non-Official). Hence Statement 2 is correct.

The non-official members are representatives of the farming community and usually have an
active association with the farming community. 13 7
2
25
38
CACP submits its recommendations to the government in the form8of6Price Policy Reports every five
year. Hence Statement 1 is incorrect. -
o m
il.c
ma
Commodities covered

@ g
05
26 of 23 commodities, which comprise
As of now, CACP recommends MSPs
1
5
m r7
- maize, sorghum, pearl millet, barley and ragi)
7 cereals (paddy, wheat,
oy
j oyR
i t un (gram, tur, moong, urad, lentil)
5 pulses

Mr
7 oilseeds (groundnut, rapeseed-mustard, soyabean, seasmum, sunflower, safflower,
nigerseed)

4 commercial crops (copra, sugarcane, cotton and raw jute).

40 Consider the following statements regarding United Nations Commission on International Trade Law
(UNCITRAL)
1. It is the core legal body of the United Nations system in the field of international trade law,
specializing in commercial law reform.
2. Its business is the modernization and harmonization of rules on international business.

Which of the statements given above is/are correct?


A. 1 only
B. 2 only

prelims.insightsonindia.com 27
© Insights Active Learning | All rights reserved - 156493. You may not reproduce, distribute or exploit the contents in any form without
written permission by copyright owner. Copyright infringers may face civil and criminal liability
Total Marks : 200
Online Prelims TEST - 25 (SUBJECT)
( InsightsIAS Mock Test Series for UPSC Preliminary Exam 2020 ) Mark Scored : 31.33

C. Both 1 and 2
D. Neither 1 nor 2

Your Answer :
Correct Answer : C

Answer Justification :

Both the statements are correct.

United Nations Commission on International Trade Law (UNCITRAL) is the core legal body
of the United Nations system in the field of international trade law, specializing in
commercial law reform.

Its business is the modernization and harmonization of rules on international business.

13 fair, and
In order to increase trade opportunities worldwide, UNCITRAL is formulating modern,
7
harmonized rules on commercial transactions. These include: 2
8 25
i. Conventions, model laws and rules which are acceptable worldwide
8 63
-
ii. Legal and legislative guides and recommendations of great o mpractical value
l .c
iii. Updated information on case law and enactmentsm aofi uniform commercial law
@ g
6 05
iv. Technical assistance in law reform projects
5 12
v. Regional and national seminars
m r7on uniform commercial law
o y-
41 Consider the following
j oy Rstatements regarding Asia-Pacific Institute for Broadcasting Development
uninter-governmental organization servicing country of the UN-ESCAP in the field of
(AIBD)
i t
Mr media development.
1. It is a regional
electronic
2. India is not a full member of this institute.

Which of the statements given above is/are correct?


A. 1 only
B. 2 only
C. Both 1 and 2
D. Neither 1 nor 2

Your Answer :
Correct Answer : A

Answer Justification :

The Asia-Pacific Institute for Broadcasting Development (AIBD), established in 1977


under the auspices of UNESCO, is a unique regional inter-governmental organisation
servicing countries of the United Nations Economic and Social Commission for Asia and

prelims.insightsonindia.com 28
© Insights Active Learning | All rights reserved - 156493. You may not reproduce, distribute or exploit the contents in any form without
written permission by copyright owner. Copyright infringers may face civil and criminal liability
Total Marks : 200
Online Prelims TEST - 25 (SUBJECT)
( InsightsIAS Mock Test Series for UPSC Preliminary Exam 2020 ) Mark Scored : 31.33

the Pacific (UN-ESCAP) in the field of electronic media development. It is hosted by the
Government of Malaysia and the secretariat is located in Kuala Lumpur.

The 26 full members of AIBD are the governments of the following countries: Afghanistan,
Bangladesh, Bhutan, Brunei, China, Cambodia, Fiji, France, India, Indonesia, Iran, Republic of
Korea, Laos, Malaysia, Maldives, Micronesia, Myanmar, Nepal, Pakistan, Papua New Guinea,
Philippines, Samoa, Singapore, Sri Lanka, Thailand, Vietnam. Hence statement 2 is incorrect.

The International Telecommunication Union (ITU), the United Nations Development Programme
(UNDP), and the United Nations Educational, Scientific and Cultural Organization (UNESCO) are
founding organizations of the Institute and they are non-voting members of the General Conference.

42 Consider the following statements regarding Golconda Fort


1. It was originally known as Mankal, and built on a hilltop in the year 1143.
2. The fort was built by the Kakatiya dynasty.

1 3
Which of the statements given above is/are correct?
5 27
A. 1 only 2
B. 2 only 6 38
C. Both 1 and 2 -8
o m
il.c
D. Neither 1 nor 2

Your Answer : gma


5 @
Correct Answer : C 0
126
5
Answer Justification :
r7
-m
y above are correct.
Both the statementsogiven

j oyR
un
Golconda Fort:
r i t
Moriginally
It was known as Mankal, and built on a hilltop in the year 1143. The fort was built by
the Kakatiya dynasty.

India’s one of the most outstanding citadels, the Golconda fort epitomises the sumptuous ‘Nawabi’
culture of the time.

The Golconda fort came into the possession of the Bahmani dynasty.

https://www.thehindu.com/news/cities/Hyderabad/pipeline-work-spells-doom-for-moat-at-golconda-f
ort/article30126658.ece

43 Consider the following statements regarding Karez


1. Karez’ system is a water harnessing technology that originated in Iran/Persia.
2. Karez system was built by Delhi Sultanate.

Which of the statements given above is/are correct?


A. 1 only
prelims.insightsonindia.com 29
© Insights Active Learning | All rights reserved - 156493. You may not reproduce, distribute or exploit the contents in any form without
written permission by copyright owner. Copyright infringers may face civil and criminal liability
Total Marks : 200
Online Prelims TEST - 25 (SUBJECT)
( InsightsIAS Mock Test Series for UPSC Preliminary Exam 2020 ) Mark Scored : 31.33

B. 2 only
C. Both 1 and 2
D. Neither 1 nor 2

Your Answer :
Correct Answer : A

Answer Justification :

Introduction of Kariz in India

Karez’ system (also known as Qanat) which is a water harnessing technology that originated in
Iran/Persia.

Karez system was built by Bahmani kings in 15th Century by the Bahmani kings in Bidar,
Gulbarg and Bijapur in Karnataka and also in Burhanpur in Madhya Pradesh. Hence Statement 2
is incorrect. 3 1
7
52
82
https://www.thehindu.com/news/national/karnataka/suranga-bawadi-on-world-monument-watch-list/
3
article29947854.ece 6
-8
o m
44 Consider the following statements
a il.c
1. Sangrai Dance is the traditional dance performed m
by Mog community of Mizoram.
@ g
2. Lokrang Samaroh is an extravagant festival,5organised every year by the Directorate of Culture and
Madhya Pradesh Tribal Museum. 6 0
1 2
75
r is/are correct?
mabove
Which of the statements given
-
A. 1 only
B. 2 only R oy
j oy
n
C. Both 1 and 2
itu 1 nor 2
D. Neither
Mr
Your Answer :
Correct Answer : B

Answer Justification :

Sangrai Dance

It is the traditional dance performed by Mog community of Tripura. Hence Statement 1 is


incorrect.

It is performed during the Sangrai festival falling on the last day of the month of Chaitra, which is
the last month of the Bengali Calendar Year.

It was performed for the first time at Rajpath on the occasion of the Republic day parade 2018.

Lokrang festival/Lokrang Samaroh

prelims.insightsonindia.com 30
© Insights Active Learning | All rights reserved - 156493. You may not reproduce, distribute or exploit the contents in any form without
written permission by copyright owner. Copyright infringers may face civil and criminal liability
Total Marks : 200
Online Prelims TEST - 25 (SUBJECT)
( InsightsIAS Mock Test Series for UPSC Preliminary Exam 2020 ) Mark Scored : 31.33

Lokrang Samaroh is an extravagant festival, organised every year by the Directorate of Culture and
Madhya Pradesh Tribal Museum (Adivasi Lok Kala Academy). Hence statement 2 is correct.

It is usually held in Bhopal 5 days long every year starting on Republic day.

45 Contingent Reserve Arrangement is a mechanism under

A. World Bank
B. International Monetary Fund
C. New Development Bank
D. BRICS

Your Answer : B
Correct Answer : D

Answer Justification : 1 3
527
The BRICS Contingent Reserve Arrangement (CRA) is a framework
3 82for the provision of
support through liquidity and precautionary instruments in 8 6
response to actual or potential
-
short-term balance of payments pressures. It was established in 2015 by the BRICS countries
.
Brazil, Russia, India, China and South Africa. The legal basisc om
is formed by the Treaty for the
a
Establishment of a BRICS Contingent Reserve Arrangement, i l signed at Fortaleza, Brazil on 15 July
g
2014. It entered into force upon ratification by all m
BRICS states, announced at the 7th BRICS
@
summit in July 2015. 05 6
2
7 51
46 Consider the following statements
- mr regarding Organisation of Islamic Cooperation (OIC)
y
1. It is the second largestointer-governmental organization after the United Nations
y
2. India is an observer R
state to OIC

u njo
t statements given above is/are correct?
Which ofrithe
M
A. 1 only
B. 2 only
C. Both 1 and 2
D. Neither 1 nor 2

Your Answer : B
Correct Answer : A

Answer Justification :

The Organization of Islamic Cooperation (OIC) is the second largest inter-governmental


organization after the United Nations with a membership of 57 states spread over four
continents. The Organization is the collective voice of the Muslim world. It endeavors to safeguard
and protect the interests of the Muslim world in the spirit of promoting international peace and
harmony among various people of the world.

Observer States are

prelims.insightsonindia.com 31
© Insights Active Learning | All rights reserved - 156493. You may not reproduce, distribute or exploit the contents in any form without
written permission by copyright owner. Copyright infringers may face civil and criminal liability
Total Marks : 200
Online Prelims TEST - 25 (SUBJECT)
( InsightsIAS Mock Test Series for UPSC Preliminary Exam 2020 ) Mark Scored : 31.33

Bosnia and Herzegovina (1994)

Central African Republic (1996)

Kingdom of Thailand (1998)

The Russian Federation (2005)

Turkish Cypriot State (1979)

India is not an observer state to OIC. Hence statement 2 is incorrect.

47 Which of the following is not a founder member of Organization of the Petroleum7 13


Exporting Countries
(OPEC) 5 2
8 2
63
A. Iran -8
B. Venezuela o m
C. Saudi Arabia il.c
D. Qatar
gma
5 @
0
Your Answer : D
126
Correct Answer : D 5
r7
Answer Justification : y -m
y Ro
u nanjoagreement
The Organization of the Petroleum Exporting Countries (OPEC) was founded in Baghdad, Iraq, with

rit
the signing of in September 1960 by five countries namely Islamic Republic of Iran,
Iraq, M
Kuwait, Saudi Arabia and Venezuela. They were to become the Founder Members of the
Organization.

The OPEC Statute distinguishes between the Founder Members and Full Members - those
countries whose applications for membership have been accepted by the Conference.

48 Consider the following statements regarding Inter-Parliamentary Union


1. It is a global organization of national parliaments.
2. Indian Parliament is also a member to this union.

Which of the statements given above is/are correct?


A. 1 only
B. 2 only
C. Both 1 and 2
D. Neither 1 nor 2

prelims.insightsonindia.com 32
© Insights Active Learning | All rights reserved - 156493. You may not reproduce, distribute or exploit the contents in any form without
written permission by copyright owner. Copyright infringers may face civil and criminal liability
Total Marks : 200
Online Prelims TEST - 25 (SUBJECT)
( InsightsIAS Mock Test Series for UPSC Preliminary Exam 2020 ) Mark Scored : 31.33

Your Answer : C
Correct Answer : C

Answer Justification :

Both the statements are correct.

The IPU is the global organization of national parliaments.

What began in 1889 as a small group of parliamentarians, dedicated to promoting peace through
parliamentary diplomacy and dialogue, has since grown into a truly global organization of national
parliaments. Today, our membership inches ever closer to being universal, with 179 Member.

49 Consider the following statements regarding Jan Sampark programme


1. The Programme enables the public to have interaction with its officials and staff for seeking
information related to adoption.
2. It is an annual programme to facilitate adoption. 7 13
3. The central adoption resource authority (CARA) is the nodal authority. 25
2
3 8
Which of the statements given above is/are correct? - 86
A. 1, 2 and 3
.c om
B. 2 and 3 only
a il
C. 1 and 3 only
@ gm
D. 1 and 2 only 5 0
6
5 12
Your Answer : 7
Correct Answer : C
- mr
y
y :
Answer Justification Ro
u njo
it Programme enables the public to have interaction with its officials and staff for
Jan Sampark
r
M information related to adoption. Hence, statement 1 is correct.
seeking

It is a monthly programme to facilitate adoption. Hence, statement 2 is incorrect.

The Central Adoption Resource Authority (CARA) under MoWCD is the nodal authority. Hence,
statement 3 is correct.

50 Consider the following statements regarding Asia–Europe Meeting


1. It is an Asian–European forum to enhance relations and various forms of cooperation between its
partners.
2. The focus of the dialogue forum is limited to political and economic issues

Which of the statements given above is/are correct?


A. 1 only
B. 2 only
C. Both 1 and 2

prelims.insightsonindia.com 33
© Insights Active Learning | All rights reserved - 156493. You may not reproduce, distribute or exploit the contents in any form without
written permission by copyright owner. Copyright infringers may face civil and criminal liability
Total Marks : 200
Online Prelims TEST - 25 (SUBJECT)
( InsightsIAS Mock Test Series for UPSC Preliminary Exam 2020 ) Mark Scored : 31.33

D. Neither 1 nor 2

Your Answer :
Correct Answer : A

Answer Justification :

The Asia–Europe Meeting (ASEM) is an Asian–European political dialogue forum to


enhance relations and various forms of cooperation between its partners. It was officially
established on 1 March 1996 at the 1st ASEM Summit (ASEM1) in Bangkok, Thailand.

The main components of the ASEM Process rest on the following 3 pillars:

Political Pillar

1 3
27
Economic & Financial Pillar
2 5
26is incorrect.
Social, Cultural & Educational Pillar. Hence statement8
38
-
.c om
51 Consider the following statements regarding Cartagena a l
iBiosafety Protocol
gm
1. It was negotiated under the aegis of the United Nations Framework Convention on Climate Change.
@
2. India is a party to the Protocol. 05 6
2
51
7 is/are correct?
r
Which of the statements given above
A. 1 only
y -m
B. 2 only
R o
C. Both 1 ando2
j y
un1 nor 2
D. Neither
it
M r
Your Answer : C
Correct Answer : B

Answer Justification :

Cartagena Biosafety Protocol (CPB): It was negotiated under the aegis of the Convention
on Biological Diversity (CBD) and adopted in 2000. Hence statement 1 is incorrect.

India is a party to the Protocol. The main objective of the Protocol is to ensure safe transfer,
handling and use of living modified organisms (LMOs) resulting from modern biotechnology that
may have adverse effect on the Environment.

52 Consider the following statements regarding Central Zoo Authority


1. It is a statutory body was established under the provisions of the Environment (Protection) Act,
1986
2. The objective is to enforce minimum standards and norms for upkeep and healthcare of animals in

prelims.insightsonindia.com 34
© Insights Active Learning | All rights reserved - 156493. You may not reproduce, distribute or exploit the contents in any form without
written permission by copyright owner. Copyright infringers may face civil and criminal liability
Total Marks : 200
Online Prelims TEST - 25 (SUBJECT)
( InsightsIAS Mock Test Series for UPSC Preliminary Exam 2020 ) Mark Scored : 31.33

the Indian zoos.

Which of the statements given above is/are correct?


A. 1 only
B. 2 only
C. Both 1 and 2
D. Neither 1 nor 2

Your Answer : B
Correct Answer : B

Answer Justification :

The Central Zoo Authority with its headquarters in New Delhi was established in 1992,
under the provisions of the Wild Life (Protection) Act, 1972 to oversee the functioning of zoos
in the country with the view to enhance their role in conservation. Hence statement 1 is
incorrect. 13 7
2
The main objective of the Central Zoo Authority is to enforce minimum 8 25
standards and
6 3
norms for upkeep and healthcare of animals in the Indian zoos8and to control the mushrooming
of ill-conceived, ill planned zoos, to monitor and evaluate them
-
existing zoos and to suggest ways and
o can be transferred into potent
.cthey
means for the improvement of zoos in the country so that
a i l
centers for ex-situ conservation of endangered wild fauna.
@ gm
0 5
26 Rashtriya Mahila Kosh
53 Consider the following statements regarding
1
75Ministry of Women & Child Development.
1. It is an autonomous body underrthe
- m micro-credit to poor women through intermediary organizations.
2. The main objective is to provide

R oy
j oy
Which of the statements given above is/are correct?
A. 1 onlyun
B. 2 r it
only
M
C. Both 1 and 2
D. Neither 1 nor 2

Your Answer :
Correct Answer : C

Answer Justification :

Both the statements given above are correct.

Rashtriya Mahila Kosh (RMK), established in 1993 is a national level organization as an


autonomous body under the aegis of the Ministry of Women and Child Development, for
socio-economic empowerment of women. The operating model currently followed by RMK is
that of a facilitating agency wherein RMK provides loans to NGO-MFIs termed as Intermediary
Organizations (IMO) which on-lend to Self Help Groups (SHGs) of women.

Rashtriya Mahila Kosh (RMK) is a society, registered under the Societies Registration Act, 1860 and

prelims.insightsonindia.com 35
© Insights Active Learning | All rights reserved - 156493. You may not reproduce, distribute or exploit the contents in any form without
written permission by copyright owner. Copyright infringers may face civil and criminal liability
Total Marks : 200
Online Prelims TEST - 25 (SUBJECT)
( InsightsIAS Mock Test Series for UPSC Preliminary Exam 2020 ) Mark Scored : 31.33

an apex micro-finance organization established in 1993. The main objective of RMK is to


provide micro-credit to poor women through intermediary organizations (IMO), which
includes Section 25 companies, NGOs among others for various livelihood support and income
generating activities at concessional terms in a client-friendly procedure to bring about their socio-
economic development. The target beneficiaries are entrepreneurs from different economic
activities ranging from traditional and modern handicraft to small business such as petty shop, etc.
The loans are sanctioned through various schemes of RMK viz., main loan scheme, loan promotion
scheme, etc.

54 Consider the following statements regarding National Adaptation Fund for Climate Change (NAFCC)
1. It is a Centrally Sponsored Scheme to support concrete adaptation activities which are not covered
under on-going schemes.
2. National Bank for Agriculture and Rural Development (NABARD) as National Implementing Entity
(NIE)

Which of the statement above is/are correct? 1 3


A. 1 only
5 27
2
B. 2 only
6 38
C. Both 1 and 2
-8
D. Neither 1 nor 2
o m
il.c
Your Answer : C
gma
Correct Answer : B
5 @
0
Answer Justification : 126
5
r7
- mClimate Change (NAFCC) is a central sector scheme under
National Adaptation Fund for
y
implementation in the o
y R 12th Five Year Plan with National Bank for Agriculture and Rural
n jo
Development (NABARD) as National Implementing Entity (NIE). Hence statement 1 is

it u
incorrect. The overall aim of the fund is to support concrete adaptation activities which are not
coveredrunder on-going activities through the schemes of state and central government, that reduce
M
the adverse impact of climate change facing communities, sectors and states.

55 Consider the following statements regarding Contingency Fund of India


1. Contingency Fund is created as an imprest account to meet some urgent or unforeseen expenditure
of the government.
2. This fund is at the disposal of the President.
3. The corpus of the Fund authorized by the Parliament, at present, is ₹ 5000 crore.

Which of the statements given above is/are correct?


A. 1 and 2 only
B. 2 and 3 only
C. 1 and 3 only
D. 1, 2 and 3

Your Answer : A

prelims.insightsonindia.com 36
© Insights Active Learning | All rights reserved - 156493. You may not reproduce, distribute or exploit the contents in any form without
written permission by copyright owner. Copyright infringers may face civil and criminal liability
Total Marks : 200
Online Prelims TEST - 25 (SUBJECT)
( InsightsIAS Mock Test Series for UPSC Preliminary Exam 2020 ) Mark Scored : 31.33

Correct Answer : A

Answer Justification :

Contingency Fund is created as an imprest account to meet some urgent or unforeseen


expenditure of the government.

This fund was constituted by the government under Article 267 of the Constitution of India. This
fund is at the disposal of the President.

Parliamentary approval for such expenditure and for withdrawal of an equivalent amount from the
Consolidated Fund is subsequently obtained and the amount spent from Contingency Fund is
subsequently recouped to the Fund. The corpus of the Fund authorised by the Parliament, at
present, is ₹ 500 crore. Hence statement 3 is incorrect.

56 Financial Stability Board is established under the aegis of


1 3
5 27
A. G20 2
B. World Bank 6 38
C. International Monetary Fund (IMF) -8
o m
il.c
D. Bank for International Settlements

Your Answer : D gma


5 @
Correct Answer : A
0
126
5
Answer Justification :
r7
-m
oy (FSB) was established in 2009 under the aegis of G20 by bringing
Financial Stability Board
R
j oy
together the national authorities, standard setting bodies and international financial institutions for

u n
addressing vulnerabilities and developing and implementing strong regulatory, supervisory and
t
ri in the interest of financial stability. India is an active member of the FSB
other policies
M
having three seats in its Plenary.

57 Consider the following statements regarding National Youth Corps


1. It is implemented by the Nehru Yuva Kendra Sangathan, Ministry of Youth affairs and Sports.
2. The selection of volunteers is done by a selection committee, headed by District Collector/Deputy
Commissioner of the concerned district.
3. Young in the age group of 16-18 years would be eligible to become volunteers.

Which of the statements given above is/are correct?


A. 1 and 2 only
B. 2 and 3 only
C. 1 and 3 only
D. 1, 2 and 3

Your Answer :

prelims.insightsonindia.com 37
© Insights Active Learning | All rights reserved - 156493. You may not reproduce, distribute or exploit the contents in any form without
written permission by copyright owner. Copyright infringers may face civil and criminal liability
Total Marks : 200
Online Prelims TEST - 25 (SUBJECT)
( InsightsIAS Mock Test Series for UPSC Preliminary Exam 2020 ) Mark Scored : 31.33

Correct Answer : A

Answer Justification :

National Youth Corps

The National Youth Corps (NYC) is a scheme implemented by the Nehru Yuva Kendra
Sangathan, Ministry of Youth affairs and Sports, Government of India, to tap the potential
of the youth and channeling their energy towards nation building.

The Scheme of National Youth Corps (NYC) was launched in 2010-11 and the same is being
implemented through NYKS. The main objectives of the Scheme are as follows: to set up a group of
disciplined and dedicated youth who have the inclination and spirit to engage in the task of nation-
building; to facilitate the realization of inclusive growth (both social and economic); to act as group
modulators and peer group educators; and to act as role models for the younger cohort, especially
towards enhancement of public ethics, probity and dignity of labour.
1 3
Eligibility
5 27
2
6 38
- 8 would be eligible to become
Young men and women falling in the age group of 18-29
volunteers. Hence statement 3 is incorrect. om
a il.c
Education qualification: - Minimum class@ gm
10 th pass
5
2 60
51given to the following:
Preference in selection may7be
- mr
y
ohaving
y R
Candidates higher educational qualification and basic knowledge of computer
n j o
application.
i t u
Mr
Preferably, Candidates having the android mobile phone and basic knowledge of
operating different Apps pertaining to e - banking /Dgidhan etc.

Members of NYKS affiliated youth clubs.

Students who have enrolled themselves as a regular student are not eligible for deployment
under the scheme, keeping in view of the nature of their full-time assignment.

Participation of weaker sections such as SC/ST community will be encouraged and 50:50
gender ratios should be maintained to the extent possible.

The minimum qualification for NYC volunteers is Class-X passed and they were initially paid an
honorarium of ₹ 2,500/- per month, but the same was increased to ₹ 5,000/- per month from 2016.

prelims.insightsonindia.com 38
© Insights Active Learning | All rights reserved - 156493. You may not reproduce, distribute or exploit the contents in any form without
written permission by copyright owner. Copyright infringers may face civil and criminal liability
Total Marks : 200
Online Prelims TEST - 25 (SUBJECT)
( InsightsIAS Mock Test Series for UPSC Preliminary Exam 2020 ) Mark Scored : 31.33

The selection of NYC volunteers is done by a selection committee, headed by District


Collector/Deputy Commissioner of the concerned district.

58 Consider the following statements regarding Controller General of Accounts (CGA)


1. Controller General of Accounts (CGA) is an office in the Department of Economic Affairs, Ministry of
Finance.
2. CGA is the Principal Accounting Adviser to Government of India and is responsible for establishing
and maintaining a technically sound Management Accounting System.
3. Under Article 150 of the Constitution, the Annual Appropriation Accounts (Civil) and Union Finance
Accounts are submitted to Parliament on the advice of Controller General of Accounts.

Which of the statements given above is/are correct?


A. 1 and 2 only
B. 2 and 3 only
C. 1 and 3 only
D. 1, 2 and 3 1 3
5 27
2
Your Answer :
6 38
Correct Answer : A
-8
o m
Answer Justification :
a il.c
gm
The Controller General of Accounts (CGA), in the Department of Expenditure, is the Principal
@is responsible for establishing and maintaining a
5and
Accounting Adviser to Government of India 0
1
technically sound Management Accounting26 System. The Office of CGA prepares monthly and annual
r
analysis of expenditure, revenues,75
borrowings and various fiscal indicators for the Union
m
y-
Government.
o
The Public Financial
j oy RManagement System (PFMS) is a web-based online software application
n
designed, developed, owned and implemented by the CGA with the aim to provide a sound public
tu
financialrimanagement system by establishing a comprehensive payment, receipt and accounting
M
network.

Under Article 150 of the Constitution, the Annual Appropriation Accounts (Civil) and
Union Finance Accounts are submitted to Parliament on the advice of Comptroller and
Auditor General of India. Hence statement 3 is incorrect.

59 Consider the following statements regarding Financial Intelligence Unit-India (FIU-IND)


1. It is the central national agency for receiving, processing, analyzing and disseminating information
relating to suspect financial transactions.
2. It is an independent body reporting to the Economic Intelligence Council (EIC) headed by the
Finance Minister.

Which of the statements given above is/are correct?


A. 1 only
B. 2 only
C. Both 1 and 2

prelims.insightsonindia.com 39
© Insights Active Learning | All rights reserved - 156493. You may not reproduce, distribute or exploit the contents in any form without
written permission by copyright owner. Copyright infringers may face civil and criminal liability
Total Marks : 200
Online Prelims TEST - 25 (SUBJECT)
( InsightsIAS Mock Test Series for UPSC Preliminary Exam 2020 ) Mark Scored : 31.33

D. Neither 1 nor 2

Your Answer :
Correct Answer : C

Answer Justification :

Financial Intelligence Unit-India (FIU-IND) is the central national agency for receiving,
processing, analyzing and disseminating information relating to suspect financial
transactions.

FIU-IND was established by the Government of India in 2004 for coordinating and strengthening
collection and sharing of financial intelligence through an effective national, regional and global
network to combat money laundering, related crimes and terrorist financing.

It is an independent body reporting to the Economic Intelligence Council (EIC) headed by


1 3
the Finance Minister. For administrative purposes, FIU-IND is under the Department of
Revenue, Ministry of Finance. 2 7
2 5
6 38
8
60 Consider the following statements regarding Pradhan Mantri Vaya-Vandana Yojana (PMVVY)
m
o exclusively for the senior citizens
1. It is a Pension Scheme announced by the Government of India
l .c
aged 60 years and above.
m ai
g
2. The Scheme is being implemented through Pension Fund Regulatory and Development Authority.
0 5@
26 correct?
Which of the statements given above is/are
1
A. 1 only
r 75
B. 2 only
y -m
Ro
C. Both 1 and 2
D. Neither 1 nory 2

u njo
r it :
Your Answer
M
Correct Answer : A

Answer Justification :

Pradhan Mantri Vaya Vandana Yojana (PMVVY) is a Pension Scheme announced by the
Government of India exclusively for the senior citizens aged 60 years and above which is
available from 4th May, 2017 to 31st March, 2020.

Government launched the Pradhan Mantri Vyay Vandana Yojana (PMVVY) to protect elderly persons
aged 60 years and above against a future fall in their interest income due to the uncertain market
condition, as also to provide social security in old age. The Scheme is being implemented
through LIC of India. Hence statement 2 is incorrect. It provides an assured return of 8 per
cent per annum payable monthly for 10 years.

61 Consider the following statements regarding National Investment Fund


1. It was constituted to channelize the proceeds from disinvestment of Central Public Sector

prelims.insightsonindia.com 40
© Insights Active Learning | All rights reserved - 156493. You may not reproduce, distribute or exploit the contents in any form without
written permission by copyright owner. Copyright infringers may face civil and criminal liability
Total Marks : 200
Online Prelims TEST - 25 (SUBJECT)
( InsightsIAS Mock Test Series for UPSC Preliminary Exam 2020 ) Mark Scored : 31.33

Enterprises.
2. 25 per cent of the annual income of the NIF was to be used for financing selected social sector
schemes.
3. 75 per cent of the annual income of NIF was to be used to meet the capital investment requirements
of profitable and revisable PSUs.

Which of the statements given above is/are correct?


A. 1 only
B. 2 and 3 only
C. 1 and 3 only
D. 1, 2 and 3

Your Answer :
Correct Answer : A

Answer Justification :
1 3
National Investment Fund 527
2
38
Government constituted the National Investment Fund (NIF) in 2005
- 86into which the proceeds from
disinvestment of Central Public Sector Enterprises were to be
o mchannelized. The corpus of NIF was
a il.cSelected
to be of a permanent nature and NIF was to be professionally
returns to the Government, without depleting the corpus.
managed to provide sustainable
Public Sector Mutual Funds,
gm
namely UTI Asset Management Company Ltd.,@SBI Funds Management
6 05
12 Management Company Ltd. were entrusted with the
Private Ltd. and LIC Mutual Fund Asset
management of the NIF corpus.r75
-m
As per this Scheme, o 75y per cent of the annual income of the NIF was to be used for
financing selected
j oy Rsocial sector schemes which promote education, health and
employment.nThe residual 25 per cent of the annual income of NIF was to be used to meet
rituinvestment requirements of profitable and revisable PSUs. Hence statement 2
the capital
M
and 3 are incorrect.

62 Consider the following statements regarding Investor Education and Protection Fund (IEPF)
1. The Companies Act provides for establishment of Investor Education and Protection Fund (IEPF).
2. It was established for promoting Investor Awareness and protecting their interests.
3. The amount of dividend, matured deposits, etc., which remained unpaid/unclaimed for a period of
seven years are to be transferred to IEPF.

Which of the statements given above is/are correct?


A. 1 only
B. 2 and 3 only
C. 1 and 3 only
D. 1, 2 and 3

Your Answer : B
Correct Answer : D

prelims.insightsonindia.com 41
© Insights Active Learning | All rights reserved - 156493. You may not reproduce, distribute or exploit the contents in any form without
written permission by copyright owner. Copyright infringers may face civil and criminal liability
Total Marks : 200
Online Prelims TEST - 25 (SUBJECT)
( InsightsIAS Mock Test Series for UPSC Preliminary Exam 2020 ) Mark Scored : 31.33

Answer Justification :

Investor Education and Protection Fund

The Companies Act provides for establishment of Investor Education and Protection Fund
(IEPF) for promoting Investor Awareness and protecting their interests. The amount of
dividend, matured deposits, matured debentures, application money, etc., which remained
unpaid/unclaimed for a period of seven years from the date they first become due for
payment, are transferred to IEPF.

Section 205 C of the Companies Act, 1956 did not allow the refund of such amount to individual
once it is transferred to IEPF. Section 125 of Companies Act, 2013 allows refund of unpaid amounts
transferred to IEPF. Such refunds are to be made by the Investor Education and Protection Fund
Authority under Section 125 of the Act.

IEPF Authority was made functional from 2016. The IEPF Authority is also entrusted with the
3
responsibilities of carrying out investor awareness activities using the amounts transferred to IEPF.
2 71
Investor Awareness Programmes (IAPs) are organized in association with 2 5 professional
three
8
63 of Company Secretaries
institutes namely; (a) Institute of Chartered Accounts of India, (b) Institute
8
-
of India and (c) Institute of Cost Accounts of India. In addition, investor awareness activities like
Crawler messages on Doordarshan through Prasar Bharati o m
and airing jingles on All India Radio for
l .c
the period of 90 days to create and increase investor’siawareness were taken up.
gma
0 5@
1 26
r 75
- m regarding Serious Fraud Investigation Office (SFIO)
63 Consider the following statements
y
1. The Companies Act, R o accords statutory status to SFIO.
2013
2. The main function y
jo is to investigate corporate frauds of serious and complex nature.
n
ri tu
Which Mof the statements given above is/are correct?
A. 1 only
B. 2 only
C. Both 1 and 2
D. Neither 1 nor 2

Your Answer : B
Correct Answer : C

Answer Justification :

Serious Fraud Investigation Office

The Serious Fraud Investigation Office (SFIO) was set up in 2003. The Companies Act, 2013,
interalia, has accorded statutory status to SFIO and its functions and powers have been
enhanced substantially with number of enabling provisions in the Act and it was
established under Section 211 of the Companies Act, 2013.

prelims.insightsonindia.com 42
© Insights Active Learning | All rights reserved - 156493. You may not reproduce, distribute or exploit the contents in any form without
written permission by copyright owner. Copyright infringers may face civil and criminal liability
Total Marks : 200
Online Prelims TEST - 25 (SUBJECT)
( InsightsIAS Mock Test Series for UPSC Preliminary Exam 2020 ) Mark Scored : 31.33

The main function of SFIO is to investigate corporate frauds of serious and complex
nature.

It takes up investigation of frauds characterized by complexity, and having inter-departmental and


multi-disciplinary ramifications, substantial involvement of public interest to be judged by size of
either monetary appropriation or the number of persons affected and the possibility of

investigations leading to, or contributing towards a clear improvement in systems, laws or


procedures. Investigations are carried out by a multi-disciplinary team which includes experts from
the field of accountancy, forensic auditing, taxation, customs and central excise,

information technology, capital market, financial transaction (including banking) and enforcement
agencies like Central Bureau of Investigation (CBI), Intelligence Bureau (IB) and Enforcement
Directorate.

64 Consider the following statements regarding Consumer Welfare Fund


3
1protect
2 7
1. It was created with the objective of providing financial assistance to promote and the
welfare of the consumer
8 25
2. It was set up by Department of Consumer Affairs. 63
-8
Which of the statements given above is/are correct?
.c om
A. 1 only a il
B. 2 only
@ gm
C. Both 1 and 2
6 05
D. Neither 1 nor 2 2 1
5
Your Answer : C m r7
-
Correct Answer : A oy
j oyR
u n
Answer Justification :
i t
Mr
Consumer Welfare Fund

The Central Excise and Salt Act, 1944 was amended in 1991 to enable the Central government to
create a Consumer Welfare Fund into which unclaimed central excise revenues not refundable to
the manufacturers would be credited annually.

The Consumer Welfare Fund was created in 1992 with the objective of providing financial
assistance to promote and protect the welfare of the consumer, create consumer
awareness and strengthen the voluntary consumer movement in the country, particularly
in rural areas.

The Department of Consumer Affairs operates the fund, set up by the Department of
Revenue under the Central Excise and Salt Act, 1944. Hence statement 2 is incorrect.
Under its Rules, since revised in 2014, any agencies/organizations engaged in consumer welfare
activities for at least a period of five years and registered under the Companies Act, Societies
Registration Act, Cooperative Societies Act or any other law for the time being in force are eligible
for financial assistance from the Fund. A sum of ₹ 26.23 crore was available in the Fund in March

prelims.insightsonindia.com 43
© Insights Active Learning | All rights reserved - 156493. You may not reproduce, distribute or exploit the contents in any form without
written permission by copyright owner. Copyright infringers may face civil and criminal liability
Total Marks : 200
Online Prelims TEST - 25 (SUBJECT)
( InsightsIAS Mock Test Series for UPSC Preliminary Exam 2020 ) Mark Scored : 31.33

2017. An amount of ₹ 11.65 crore was utilized by December 2017 from the Fund.

65 Consider the following statements regarding Mid-Day Meal Scheme


1. It is implemented by the Ministry of Health and Family Welfare.
2. The Scheme covers students of primary and upper primary classes in the government
schools/schools aided by government.

Which of the statements given above is/are correct?


A. 1 only
B. 2 only
C. Both 1 and 2
D. Neither 1 nor 2

Your Answer : B
Correct Answer : B
1 3
Answer Justification : 527
2
38
The Mid-Day Meal Scheme is implemented by the Ministry of
- 86Human Resource
Development. Hence statement 1 is incorrect. The Scheme covers students of primary and
upper primary classes in the government schools/schools.c om by government and the schools run
aided
l
ati100 grams for primary stage and at 150
by local bodies. Foodgrains are supplied free of cost a
gmday where cooked/processed hot meal is being
grams for upper primary stage per child per school
@
served or 3 kgs 05 6
2
51
per student per month where raw7foodgrains are distributed. During 2018-19, 12.12 lakh tonnes of
m r
-
foodgrains comprising 10.25 lakh tonnes of rice and 1.87 lakh tonnes of wheat were allocated under
the Scheme. oy
j oyR
n
r it u
66 Consider the following statements regarding Annapurna Scheme
M
1. This Scheme is implemented by Ministry of Consumer Affairs, Food and Public Distribution
2. Indigent senior citizens of 65 years of age or above who are not getting pension under the National
Old Age Pension Scheme (NOAPS), are eligible for this scheme.

Which of the statements given above is/are correct?


A. 1 only
B. 2 only
C. Both 1 and 2
D. Neither 1 nor 2

Your Answer :
Correct Answer : B

Answer Justification :

Annapurna Scheme

prelims.insightsonindia.com 44
© Insights Active Learning | All rights reserved - 156493. You may not reproduce, distribute or exploit the contents in any form without
written permission by copyright owner. Copyright infringers may face civil and criminal liability
Total Marks : 200
Online Prelims TEST - 25 (SUBJECT)
( InsightsIAS Mock Test Series for UPSC Preliminary Exam 2020 ) Mark Scored : 31.33

This Scheme is implemented by the Ministry of Rural Development. Hence statement 1 is


incorrect.

Indigent senior citizens of 65 years of age or above who are not getting pension under the
National Old Age Pension Scheme (NOAPS), are provided 10 kgs. of food grains per person
per month free of cost under it.

Food grains are provided by the Department of Food and Public Distribution under the scheme at
BPL prices.

67 Consider the following statements regarding International Grains Council


1. It is an intergovernmental forum of exporting and importing countries for cooperation in wheat and
coarse grain matters.
2. India is a member of the Council and has been included in the category of importing member.

Which of the statements given above is/are correct?


1 3
A. 1 only
5 27
2
38
B. 2 only
C. Both 1 and 2 6
D. Neither 1 nor 2 -8
o m
il.c
ma
Your Answer : A
Correct Answer : A
@ g
0 5
Answer Justification :
126
5
m
International Grains Council
r7
o y-
India is a member
j oyRof the International Grains Council (IGC), an intergovernmental forum
of exportingnand importing countries for cooperation in wheat and coarse grain matters
u
itpreviously
M r
which was known as International Wheat Council up to 1995. It administers the Grains
Trade

Convention, 1995.

The IGC Secretariat, based in London since 1949, also services the Food Aid Committee,
established under the Food Aid Convention. International Grains Agreement comprises Grains
Trade Convention (GTC) and Food Aid Convention (FAC). India is a signatory to the International
Grains Agreement (IGA), 1995 and its Grain Trade Convention (GTC), 1995 which is effective from
1995. IGC has two types of members—importing and exporting members. India has been
included in the category of exporting member in July, 2003 and represented in the
meetings/sessions of the Council held from time to time. Hence statement 2 is incorrect.

68 Consider the following statements regarding Integrated Disease Surveillance Programme


1. It is a centrally sponsored scheme under NHM implemented in all states and UTs.
2. Its objective is to strengthening /maintaining a decentralized laboratory-based IT-enabled disease
surveillance system for epidemic prone diseases

prelims.insightsonindia.com 45
© Insights Active Learning | All rights reserved - 156493. You may not reproduce, distribute or exploit the contents in any form without
written permission by copyright owner. Copyright infringers may face civil and criminal liability
Total Marks : 200
Online Prelims TEST - 25 (SUBJECT)
( InsightsIAS Mock Test Series for UPSC Preliminary Exam 2020 ) Mark Scored : 31.33

Which of the statements given above is/are correct?


A. 1 only
B. 2 only
C. Both 1 and 2
D. Neither 1 nor 2

Your Answer :
Correct Answer : C

Answer Justification :

Integrated Disease Surveillance Programme

Integrated Disease Surveillance Programme (IDSP) is a centrally sponsored scheme under


NHM implemented in all states and UTs. The main objectives and strategies include
strengthening /maintaining a decentralized laboratory-based IT-enabled disease
surveillance system for epidemic prone diseases to monitor disease trends
7 13and to detect
2
and respond to outbreaks in early rising phase through trained rapid5response teams.
3 82
86 e-mail/portal. The weekly
Presently, more than 90 per cent districts report weekly data through
-
data are analyzed by SSU/DSU for disease trends. Whenever there is rising trend of illnesses, it is
.
investigated by the RRT to diagnose and control the outbreak.c om
a il
@ gm
05Arab region.
69 Consider the following statements regarding Unani
6
1. The Unani system of medicine originated
5 12 and practices relating to the promotion of positive health
in

r7
2. It is based on well-established knowledge
and prevention of disease. m
-
ynaturally
3. It emphasizes the use of
R o occurring mostly herbal medicines and some medicines of animal
origin.
j oy
n
tustatements given above is/are correct?
Which ofrithe
A.M1 and 2 only
B. 2 and 3 only
C. 1 and 3 only
D. 1, 2 and 3

Your Answer :
Correct Answer : B

Answer Justification :

Unani

The Unani system of medicine originated in Greece and passed through many countries
before establishing itself in India during the medieval period. Hence statement 1 is
incorrect. It is based on well-established knowledge and practices relating to the
promotion of positive health and prevention of disease. The fusion of traditional knowledge of
ancient civilizations like Egypt, Arabia, Iran, China, Syria and India.

prelims.insightsonindia.com 46
© Insights Active Learning | All rights reserved - 156493. You may not reproduce, distribute or exploit the contents in any form without
written permission by copyright owner. Copyright infringers may face civil and criminal liability
Total Marks : 200
Online Prelims TEST - 25 (SUBJECT)
( InsightsIAS Mock Test Series for UPSC Preliminary Exam 2020 ) Mark Scored : 31.33

It emphasizes the use of naturally occurring mostly herbal medicines and some medicines
of animals, marine and mineral origin. This system of medicine was documented in Al- Qanoon,
a medical classic, by Sheikh Bu-Ali Sina (Avicena) (980-1037) AD), in Al-Havi by Razi (850-923 AD)
and in many others books written by the Unani physicians.

70 Consider the following statements regarding Survey of India (SOI)


1. It has been assigned the role as the National Principal Mapping Agency.
2. It works under the Administrative control of Ministry of Mines

Which of the statements given above is/are correct?


A. 1 only
B. 2 only
C. Both 1 and 2
D. Neither 1 nor 2

Your Answer : A 1 3
Correct Answer : A 5 27
2
6 38
Answer Justification :
-8
m
Survey of India (SOI), the national survey and mapping
i l .co organization under the Ministry of
m a
Science and Technology. Hence statement 2 is incorrect. In its assigned role as the
g
National Principal Mapping Agency, Survey of India bears a special responsibility to ensure that
0
the country’s domain is explored and mapped 5@suitably to provide base maps for expeditious and
6
12 all resources contribute their full measure to the progress,
integrated development and ensure that
5
r7
prosperity and security of the country.
-m
oy of SOI is to maintain topographical map coverage on 1:250K, 1:50K and
The primary responsibility
R
j oy
l:25K scales. The topographical map coverage on l:250K and 1:50K scales covering the entire
n completed and the map sheets are being updated at regular intervals to depict
country hasubeen
i t
r information.
latest spatial
M
71 Consider the following statements
1. Rajasthan is endowed with the largest resources of lead-zinc ore in India.
2. Karnataka has the highest resource of Manganese Ore.
3. About 92 per cent resources of nickel are found in Odisha.

Which of the statements given above is/are correct?


A. 1 and 2 only
B. 2 and 3 only
C. 1 and 3 only
D. 1, 2 and 3

Your Answer : B
Correct Answer : C

Answer Justification :
prelims.insightsonindia.com 47
© Insights Active Learning | All rights reserved - 156493. You may not reproduce, distribute or exploit the contents in any form without
written permission by copyright owner. Copyright infringers may face civil and criminal liability
Total Marks : 200
Online Prelims TEST - 25 (SUBJECT)
( InsightsIAS Mock Test Series for UPSC Preliminary Exam 2020 ) Mark Scored : 31.33

Lead-zinc ore

Rajasthan is endowed with the largest resources of lead-zinc ore amounting to 670.34
million tonnes (89.44 per cent), followed by Andhra Pradesh 22.69 million tonnes (3.02 per
cent), Madhya Pradesh 14.84 million tonnes (1.98 per cent), Bihar 11.43 million tonnes (1.52 per
cent) and Maharashtra 9.27 million tonnes (1.24 per cent).

Manganese Ore

Statewise, Odisha tops the total resources with 44 per cent share followed by Karnataka 22
per cent, Hence statement 2 is incorrect. Madhya Pradesh 12 per cent, Maharashtra and Goa 7
per cent each, Andhra Pradesh 4 per cent and Jharkhand 2 per cent. Rajasthan, Gujarat and West
Bengal together shared the remaining about 2 per cent resources.

Nickel

As per UNFC, as in 2015, the total resources of nickel ore have been estimated at 189
million tonnes. About 92 per cent resources; i.e., 175 million tonnes are7in 13Odisha. The
2 52and Nagaland (5
remaining 8 per cent resources are distributed in Jharkhand (9 million tonnes)
8
million tonnes). Nominal resources are reported from Karnataka (0.2363million tonnes).
-8
om
il.c Guard (NSG)
72 Consider the following statements regarding National Security
a
m the Ministry of defense.
1. NSG was raised as a Federal Contingency Force under
g
@
2. It consists of selected and highly motivated personnel from the army only.
5
0
26
51is/are correct?
Which of the statements given above
7
A. 1 only
- mr
oy
B. 2 only
C. Both 1 and 2 R
D. Neithern1jo
y
nor 2
rit u
Your M
Answer : A
Correct Answer : D

Answer Justification :

National Security Guard

Terrorism, both national and international raised its head in the west during the seventies. It
manifested in many forms including hijacking of aircraft, taking of hostages, assassination of
dignitaries and others.

The need for creating a special force for executing surgical operations based on tactical intelligence
was felt in India when Operation Blue Star was carried out by the army at the Golden Temple,
Amritsar in 1984.

National Security Guard (NSG) was conceptualised and created after studying and analyzing
Special Force like SAS in the United Kingdom, GIGN in France, GSG-9 in Germany, Shar-etmatkal

prelims.insightsonindia.com 48
© Insights Active Learning | All rights reserved - 156493. You may not reproduce, distribute or exploit the contents in any form without
written permission by copyright owner. Copyright infringers may face civil and criminal liability
Total Marks : 200
Online Prelims TEST - 25 (SUBJECT)
( InsightsIAS Mock Test Series for UPSC Preliminary Exam 2020 ) Mark Scored : 31.33

in Israel and Delta Force in the USA. Accordingly, NSG was raised in October, 1985 as a
Federal Contingency Force under the MHA. It consists of selected and highly motivated
personnel from the army as well as the central armed police. Hence statements 1 and 2
are incorrect. 53 per cent of the personnel are drawn from the army while the central armed
police like BSF, CRPF and ITBP contribute 47 per cent.

NSG has a glorious heritage and is held in high esteem by the nation. Since raising, its commandos
have been employed in 114 major successful counter terrorist operations, earning

numerous awards and rewards including three Ashok Chakras, two Kirti Chakras, three Shaurya
Chakras and 10 PPMG. Nine NSG Commandos have also made the supreme sacrifice in living up to
the NSG’s motto of Sarvatra Sarvottam Suraksha.

73 Consider the following statements regarding Press Trust of India (PTI)


#118921

1to3provide efficient
1. It is India’s largest news agency.
7
2. It is a non-profit cooperative owned by the country’s newspapers with a mandate
and unbiased news to all subscribers. 2 52
8
3. It offers its news services in regional languages. 63
-8
Which of the statements given above is/are correct?
.c om
A. 1 and 2 only a il
B. 2 and 3 only
@ gm
C. 1 and 3 only
6 05
D. 1, 2 and 3 2 1
5
Your Answer : A m r7
-
Correct Answer : A oy
j oyR
u n
Answer Justification :
i t
Mr
India’s largest news agency, Press Trust of India (PTI) is a non-profit sharing cooperative
owned by the country’s newspapers with a mandate to provide efficient and unbiased news
to all subscribers. Founded in August, 1947, PTI began functioning from 1949. It offers its news
services in English and Hindi languages. Hence statement 3 is incorrect.

Bhasha is the Hindi language news service of the agency. PTI subscribers include 500 newspapers
in India and scores abroad. All major TV and radio channels in India and several abroad, including
BBC in London, receive its services.

PTI now has its own satellite delivery system through a transponder on an INSAT satellite for
reaching its services directly to subscribers anywhere in the country. Increasingly more and more
subscribers are opting for satellite reception. Photo service is delivered by satellite as well as

accessed by dial up.

74 Consider the following statements regarding Central Board of Film Certification (CBFC)

prelims.insightsonindia.com 49
© Insights Active Learning | All rights reserved - 156493. You may not reproduce, distribute or exploit the contents in any form without
written permission by copyright owner. Copyright infringers may face civil and criminal liability
Total Marks : 200
Online Prelims TEST - 25 (SUBJECT)
( InsightsIAS Mock Test Series for UPSC Preliminary Exam 2020 ) Mark Scored : 31.33

1. The first film in India, Raja Harishchandra, was produced in 1913 by Dadasaheb Phalke.
2. Films can be publicity exhibited in India only after they have been certified by the Central Board of
Film Certification.
3. Secretary to Ministry of Information and Broadcasting is the Ex-officio chairman of the board.

Which of the statements given above is/are correct?


A. 1 and 2 only
B. 2 and 3 only
C. 1 and 3 only
D. 1, 2 and 3

Your Answer :
Correct Answer : A

Answer Justification :

Central Board of Film Certification (CBFC) is a statutory body under Ministry of 1 3


527
82 the provisions of the
Information and Broadcasting, regulating the public exhibition of films under
3
Cinematograph Act, 1952. 86 -
om
a il.c
Films can be publicity exhibited in India only
@ gm after they have been certified by the Central
Board of Film Certification. Though the
6 05first film in India (Raja Harishchandra) was
produced in 1913 by Dadasaheb Phalke,
1 2 the Indian Cinematograph Act was passed and
came into effect only in 1920.7 5
- mr
R oy
j oythey were called then) were placed under police chiefs in cities of Madras,
un Lahore and Rangoon. Regional censors were independent. After Independence
Censor Boards (as
t
Bombay, iCalcutta,
Mr of regional censors was abolished and they were brought under the Bombay Board of
autonomy
Film Censors.

With implementation of Cinematograph Act, 1952, the board

was unified and reconstituted, as the Central Board of Film Censors.

Cinematograph (Certification) Rules were revised in 1983 and since then the Central Board of Film
Censors became known as the Central Board of Film Certification. The Board consists of non-
official members and a chairman (all of whom are appointed by central government) and
functions with headquarters at Mumbai. Hence statement 3 is incorrect.

75 Consider the following statements regarding India International Science Festival (IISF)
1. It is an annual event organised jointly by science and technology related Ministries and
Departments of the Government of India and Vijnana Bharati (Vibha)

prelims.insightsonindia.com 50
© Insights Active Learning | All rights reserved - 156493. You may not reproduce, distribute or exploit the contents in any form without
written permission by copyright owner. Copyright infringers may face civil and criminal liability
Total Marks : 200
Online Prelims TEST - 25 (SUBJECT)
( InsightsIAS Mock Test Series for UPSC Preliminary Exam 2020 ) Mark Scored : 31.33

2. It aims to build a strategy for inclusive advancement of Science and Technology


3. Recently Fifth India International Science Festival was held at Bengaluru.

Which of the statements given above is/are correct?


A. 2 only
B. 1 and 2 only
C. 2 and 3 only
D. 1, 2 and 3

Your Answer :
Correct Answer : B

Answer Justification :

India International Science Festival (IISF) is an annual event organised jointly by science and
technology related Ministries and Departments of the Government of India and Vijnana Bharati
(Vibha). Hence Statement 1 is correct. 13 7
2
25
6 38 Itcontribution
Objective: To instill scientific temper among the masses, showcase India’s in the field

- 8
of S&T over the years and encourage translation of its benefits to people. aims to build a strategy
for inclusive advancement of Science and Technology. Hence mStatement 2 is correct.
i l .co
a at Kolkata. Hence Statement 3 is
Fifth India International Science Festival is being held
m
incorrect. g
0 5@
1 26
r 75
76 Consider the following statements regarding Bougainville Island
1. Bougainville Island is the main
- misland of the autonomous Region of Bougainville of Papua New
Guinea.
R oy
2. It is the largest of ythe Solomon Islands archipelago.
3. The island has jo world’s largest copper deposits.
nthe
it u
M r
Which of the statements given above is/are correct?
A. 1 and 2 only
B. 2 and 3 only
C. 1 only
D. 1, 2 and 3

Your Answer : D
Correct Answer : D

Answer Justification :

Bougainville Island is the main island of the Autonomous Region of Bougainville of Papua New
Guinea. Bougainville Island was named after French navigator Louis Antoine de Bougainville.
Bougainvilleans are mostly Melanesian and the local language is Tok Pisin. Hence Statement 1 is
correct

Bougainville Island is the largest of the Solomon Islands archipelago. Hence Statement 2 is

prelims.insightsonindia.com 51
© Insights Active Learning | All rights reserved - 156493. You may not reproduce, distribute or exploit the contents in any form without
written permission by copyright owner. Copyright infringers may face civil and criminal liability
Total Marks : 200
Online Prelims TEST - 25 (SUBJECT)
( InsightsIAS Mock Test Series for UPSC Preliminary Exam 2020 ) Mark Scored : 31.33

correct

The island has the world’s largest copper deposits. Hence Statement 3 is correct

77 Consider the following statements regarding BrahMos missile


1. BrahMos missile is a medium-range ramjet supersonic cruise missile.
2. It is a joint venture of the DRDO of India and the NPOM of Russia.
3. It is single-stage missile.

Which of the statements given above is/are correct?


A. 2 only
B. 1 and 2 only
C. 1 only
D. None

Your Answer : A 1 3
Correct Answer : B
5 27
2
6 38
-8
Answer Justification :
m
i l .co missile. Hence Statement 1 is
BrahMos missile is a medium-range ramjet supersonic cruise
correct a
@ gm
It is a joint venture of the DRDO of India and
6 05the NPOM of Russia. It operates on fire and forget
principal. Hence Statement 2 is correct
5 12
7
rone
m
It is two-stage missile, the first
- being solid and the second one ramjet liquid propellant. Hence
Statement 3 is incorrect
R oy
j oy
78 Consider thetu
n
following statements regarding Scientific Utilization Through Research Augmentation -
ri
Prime Products from Indigenous Cows (SUTRA PIC)
M
1. It is led by Ministry of Animal Husbandry, Dairying and Fisheries
2. It is a programme to research on ‘indigenous’ cows, funded by multiple scientific ministries.

Which of the statements given above is/are correct?


A. 1 only
B. 2 only
C. Both 1 and 2
D. Neither 1 nor 2

Your Answer :
Correct Answer : B

Answer Justification :

Scientific Utilisation Through Research Augmentation - Prime Products from Indigenous Cows
(SUTRA PIC) is led by Department of Science and Technology. Hence Statement 1 is incorrect

prelims.insightsonindia.com 52
© Insights Active Learning | All rights reserved - 156493. You may not reproduce, distribute or exploit the contents in any form without
written permission by copyright owner. Copyright infringers may face civil and criminal liability
Total Marks : 200
Online Prelims TEST - 25 (SUBJECT)
( InsightsIAS Mock Test Series for UPSC Preliminary Exam 2020 ) Mark Scored : 31.33

It is a programme to research on ‘indigenous’ cows, funded by multiple scientific ministries. It has


the Department of Biotechnology, the Council of Scientific and Industrial Research, the Ministry for
AYUSH (Ayurveda, Unani, Siddha, and Homoeopathy) among others and the Indian Council of
Medical Research as partners. Hence Statement 2 is correct

79 Consider the following statements regarding Punjab Kinnow


1. It is pesticide-free fruit.
2. Punjab is the largest producer of kinnow in the country.

Which of the statements given above is/are correct?


A. 1 only
B. 2 only
C. Both 1 and 2
D. Neither 1 nor 2

Your Answer : 1 3
Correct Answer : C 5 27
2
6 38
Answer Justification :
-8
m
The Punjab Agri Export Corporation recently launched the
i l .co‘Punjab Kinnow’ brand at the kinnow
m a
festival in Abohar. This brand of kinnow, which is considered the ‘king fruit’ of Punjab, is also said
g
to be “pesticide-free”. Hence Statement 1 is correct
@
5
60
5 12
Punjab being the largest producer of kinnow in the country, such branding will attract more
consumers. Hence Statement 27is correct.
m r
o y-
80 Consider the followingR
j oy statements regarding GISAT-1 — Geo Imaging Satellite
1. It will be the n first of two planned Indian EO spacecraft to be placed in a geostationary orbit of
i t u
r
around 36,000 km.
2. It willMbe in a fixed spot looking over the Indian continent at all times.

Which of the statements given above is/are correct?


A. 1 only
B. 2 only
C. Both 1 and 2
D. Neither 1 nor 2

Your Answer :
Correct Answer : C

Answer Justification :

Indian Space Research Organisation (ISRO) is preparing to launch GISAT-1, a new earth
observation satellite, in the first week of March. It will be the first of two planned Indian EO
spacecraft to be placed in a geostationary orbit of around 36,000 km. Hence Statement 1
is correct

prelims.insightsonindia.com 53
© Insights Active Learning | All rights reserved - 156493. You may not reproduce, distribute or exploit the contents in any form without
written permission by copyright owner. Copyright infringers may face civil and criminal liability
Total Marks : 200
Online Prelims TEST - 25 (SUBJECT)
( InsightsIAS Mock Test Series for UPSC Preliminary Exam 2020 ) Mark Scored : 31.33

It will apparently be in a fixed spot looking over the Indian continent at all times. It will
have high-resolution cameraswhich will help to monitor any changes in borders and the overall
geographical condition of the country, etc. Hence Statement 2 is correct.

81 Consider the following statements regarding corporate train model


1. In this model, the corporation takes all the decisions of running the service– fare, food, onboard
facilities, housekeeping, complaints etc.
2. In this model, IRCTC doesn’t have full flexibility to decide the service parameters.
3. This creates the environment for enhanced service quality and user experience for the passengers.

Which of the statements given above is/are correct?


A. 2 only
B. 1 and 3 only
C. 3 only
D. 1, 2 and 3
1 3
Your Answer : D 5 27
2
Correct Answer : B
6 38
-8
Answer Justification :
o m
a
The Kashi Mahakal Express is the country’s third ‘corporate’
il.c
train after the two Tejas Express trains
g m
@
between Delhi-Lucknow and Mumbai-Ahmedabad started over the past few months.
5
60
Corporate train model is a new model
5 12being actively pushed by Indian Railways- to ‘outsource’ the
running of regular passenger’s r 7 to its PSU, the Indian Railway Catering and Tourism
trains
Corporation (IRCTC). -m
R oy
In this model, the y corporation takes all the decisions of running the service– fare, food, onboard
n j o
facilities, housekeeping, complaints etc. Indian Railways is free from these encumbrances and gets
t u
i IRCTC a pre-decided amount, being the owner of the network. Hence Statement 1 is
Mr
to earn from
correct.

In this model, IRCTC also has full flexibility to decide the service parameters and even alter
them without having to go to Railway ministry or its policies. Hence Statement 2 is incorrect.

This creates the environment for enhanced service quality and user experience for the passengers.
Hence Statement 3 is correct.

82 Consider the following statements regarding Madhav National Park


1. It is located in Uttar Pradesh
2. It is a combination of dry deciduous & dry thorn forests.
3. It is a part of the upper Vindhyan hills

Which of the statements given above is/are correct?


A. 1 and 3 only
B. 2 and 3 only

prelims.insightsonindia.com 54
© Insights Active Learning | All rights reserved - 156493. You may not reproduce, distribute or exploit the contents in any form without
written permission by copyright owner. Copyright infringers may face civil and criminal liability
Total Marks : 200
Online Prelims TEST - 25 (SUBJECT)
( InsightsIAS Mock Test Series for UPSC Preliminary Exam 2020 ) Mark Scored : 31.33

C. 3 only
D. 1, 2 and 3

Your Answer : D
Correct Answer : B

Answer Justification :

Madhav National Park is Located in Madhya Pradesh. It was named after Madho Rao Scindia, the
Maharaja of Gwalior belonging to the Scindia dynasty of the Marathas. Hence Statement 1 is
incorrect.

It is a combination of dry deciduous & dry thorn forests. It make home for different variety of
antelopes like Nilgai, Chinkara, and Deer, and carnivours such as leopard, wolf, jackal, fox, wild pig,
etc. Hence Statement 2 is correct.

It is a part of the upper Vindhyan hills. Hence Statement 3 is correct.


1 3
5 27
2
83 Consider the following statements regarding Konark Sun Temple 6 38
1. It was built by King Narasimhadeva I -8
2. The temple is included in UNESCO World Heritage Site om
.c
3. The temple is blend of Kalinga architecture ail
@ gm
05
Which of the statement given above is/are correct?
6
A. 1, 2 and 3
5 12
B. 3 only
C. 2 and 3 only m r7
D. 1, 2 and 3 o y-
j oyR
Your Answer n :D
i t u
Correct rAnswer :D
M
Answer Justification :

A plan to restore and preserve the nearly 800-year-old Konark Sun Temple in Odisha would be
drawn up soon, after a two-day conference of experts at the end of the month. It was built in the
13th century, the Konark temple was conceived as a gigantic chariot of the Sun God, with 12 pairs
of exquisitely ornamented wheels pulled by seven horses. It was built by King Narasimhadeva I, the
great ruler of Ganga dynasty. Hence Statement 1 is correct.

The temple is included in UNESCO World Heritage Site in 1984 for its architectural greatness and
also for the sophistication and abundance of sculptural work. Hence Statement 2 is correct.

The temple is perfect blend of Kalinga architecture, heritage, exotic beach and salient natural
beauty. . Hence Statement 3 is correct.

84 Consider the following statements regarding School Health Ambassador Initiative

prelims.insightsonindia.com 55
© Insights Active Learning | All rights reserved - 156493. You may not reproduce, distribute or exploit the contents in any form without
written permission by copyright owner. Copyright infringers may face civil and criminal liability
Total Marks : 200
Online Prelims TEST - 25 (SUBJECT)
( InsightsIAS Mock Test Series for UPSC Preliminary Exam 2020 ) Mark Scored : 31.33

1. The initiative has been launched as a part of Sarva Shiksha Abhiyan (SSA)
2. It will be jointly run by the Union ministries of Health and Human Resources Development.
3. Under the initiative, two teachers will be identified in every government school as ‘health and
wellness ambassadors’.

Which of the statements given above is/are correct?


A. 1 and 2 only
B. 3 only
C. 2 and 3 only
D. 1, 2 and 3

Your Answer : B
Correct Answer : C

Answer Justification :

13 awareness
The central Government launched the School Health Ambassador Initiative. To spread
7
about the preventive health aspects. 52 2
8
63
8 Statement 1 is incorrect.
The initiative has been launched as a part of Ayushman Bharat. Hence
-
It will be jointly run by the Union ministries of Health and om
.cHuman Resources Development. Hence
Statement 2 is correct. a i l
@ gm
6 05
Under the initiative, two teachers will be identified in every government school as ‘health and
12 3 is correct.
wellness ambassadors’. Hence Statement
5
m r7
y - state proposed a census of indigenous Muslim groups
85 Recently which of the following
o
j o yR
A. Kerala n
rit u
B. Telangana
MC. Gujarat
D. Assam

Your Answer : D
Correct Answer : D

Answer Justification :

Assam’s Welfare of Minorities and Development Department has announced plans to hold a census
of four communities broadly known as “Assamese Muslims” —Goriya, Moriya, Deshi, and Julha.
Hence, option D is correct.

The government plans to set up a corporation to look after the welfare of these communities who
number an estimated 16 lakh in Assam’s 3.3-crore population.

86 Consider the following statements regarding Asian Productivity Organization (APO)

prelims.insightsonindia.com 56
© Insights Active Learning | All rights reserved - 156493. You may not reproduce, distribute or exploit the contents in any form without
written permission by copyright owner. Copyright infringers may face civil and criminal liability
Total Marks : 200
Online Prelims TEST - 25 (SUBJECT)
( InsightsIAS Mock Test Series for UPSC Preliminary Exam 2020 ) Mark Scored : 31.33

1. India is a founder member of APO


2. DIPP is the nodal department for the promotion of productivity and quality in the industrial sector.

Which of the statements given above is/are correct?


A. 1 only
B. 2 only
C. Both 1 and 2
D. Neither 1 nor 2

Your Answer :
Correct Answer : C

Answer Justification :

DIPP is the nodal department for the promotion of productivity and quality in the
industrial sector. The National Productivity Council (NPC) represents India in the Tokyo
based Asian Productivity Organization (APO), of which India is a founder 1 3
member.
27
8
NPC undertakes productivity augmentation through domain specific consultancy, training, 25
6 3
- 8schemes
workshops, seminars and conferences to government, public and private sectors, productivity

o m
related research, monitoring and evaluation of various government and projects and
l.c
information dissemination through collaboration with APO.
i
gma
@
87 Diamonds can be found in
6 05
1. Cuddapah and Kurnool
5 12
7
2. Panna belt and Chhatarpur district
mr district
3. Behradin-Kodawali area in-Raipur

R oy
j oy
Which of the statements given above is/are correct?
A. 2 onlyun
B. 3 r it
only
M
C. 1 and 3 only
D. 1, 2 and 3

Your Answer : A
Correct Answer : D

Answer Justification :

Diamond occurrences are reported since prehistoric times in the country. Presently, diamond fields
of India are grouped into four regions:

1) South Indian tract of Andhra Pradesh, comprising parts of Anantapur, Cuddapah,


Guntur, Krishna, and Kurnool districts and Mahaboobnagar in Telangana;

2) Central Indian tract of Madhya Pradesh, comprising Panna belt and Chhatarpur
districts;

prelims.insightsonindia.com 57
© Insights Active Learning | All rights reserved - 156493. You may not reproduce, distribute or exploit the contents in any form without
written permission by copyright owner. Copyright infringers may face civil and criminal liability
Total Marks : 200
Online Prelims TEST - 25 (SUBJECT)
( InsightsIAS Mock Test Series for UPSC Preliminary Exam 2020 ) Mark Scored : 31.33

3) Behradin-Kodawali area in Raipur district and Tokapal, Dugapal, etc. areas in Bastar
district of Chhattisgarh; and

4) Eastern Indian tract mostly of Odisha, lying between Mahanadi and Godavari valleys.

As per the UNFC system as in 2015 all India resources of diamond are placed at around 31.84
million carats. Out of these, 0.96 million carats are placed under reserves category and 30.87
million carats under remaining resources category. By grades, about 2.38 per cent resources are of
gem variety, 2.64 per cent of industrial variety and bulk of the resources (95 per cent) are placed
under unclassified category. By states, Madhya Pradesh accounts for about 90.18 per cent
resources followed by Andhra Pradesh 5.73 per cent and Chhattisgarh 4.10 per cent.

88 Consider the following statements


1. Tamilnadu is the leading state in area under cultivation of coconut and its production.
2. The Lakshadweep coconuts are the highest oil content nuts in the world.

1 3
Which of the statements given above is/are correct?
5 27
A. 1 only 2
B. 2 only 6 38
C. Both 1 and 2 -8
o m
il.c
D. Neither 1 nor 2

Your Answer : gma


5 @
Correct Answer : B 0
126
5
Answer Justification :
r7
-m
y in India are Kerala, Tamilnadu, Karnataka and Andhra Pradesh.
Major coconut growingostates
R
joy statement 1 is incorrect.
Among them Kerala is the leading state in area under cultivation of coconut and its
production. n
Hence
rit u
M
Coconut is the only major crop with a production of 60 million nuts per year. The area under
cultivation is about 2,689 hectare. Lakshadweep coconut is branded as an organic product.

In Lakshadweep, coconut productivity per hectare is 20,600 and average yield per palm per year is
82 coconuts. The Lakshadweep coconuts are the highest oil content nuts in the world (82
per cent).

89 Consider the following statements regarding Rashtriya Arogya Nidhi


1. It was set up to provide financial assistance to patients, living below poverty line and who are
suffering from major life-threatening diseases.
2. It was set up as a statutory body under Ministry of Health and Family Welfare.

Which of the statements given above is/are correct?


A. 1 only
B. 2 only
C. Both 1 and 2

prelims.insightsonindia.com 58
© Insights Active Learning | All rights reserved - 156493. You may not reproduce, distribute or exploit the contents in any form without
written permission by copyright owner. Copyright infringers may face civil and criminal liability
Total Marks : 200
Online Prelims TEST - 25 (SUBJECT)
( InsightsIAS Mock Test Series for UPSC Preliminary Exam 2020 ) Mark Scored : 31.33

D. Neither 1 nor 2

Your Answer : A
Correct Answer : A

Answer Justification :

The Rashtriya Arogya Nidhi (RAN) has been set up vide Resolution No. F-7-2/96-Fin-II
dated 13/1/1997 and registered under the Societies Registration Act, 1860, as a Society.
Hence statement 2 is incorrect. The RAN was set up to provide financial assistance to patients,
living below poverty line and who are suffering from major life-threatening diseases, to receive
medical treatment at any of the super speciality Hospitals/Institutes or other Government hospitals.
The financial assistance to such patients is released in the form of ‘one-time grant’, which is
released to the Medical Superintendent of the Hospital in which the treatment has been/is being
received.

Rashtriya Arogya Nidhi (RAN) was set up in 1997 to provide financial assistance 1 to3the patients
7
2 to receive medical
living below poverty line, who are suffering from major life-threatening diseases,
2 5
6 38 theistreatment
treatment at Government hospitals. The financial assistance to such patients released in the form
of “one-time grant” to the Medical Superintendent of the hospital in
- 8 which is being
received. Under the scheme central government also provides
o mgrant-in-aid to states/union
il.c
territories (with legislature) to set up State Illness Assistance Fund (SIAF) to the extent of 50 per
cent of contribution made by state government/unionaterritories.
@ gm
0 5
26 National Deworming Day
90 Consider the following statements regarding
1
r 75
1. It was initiated to combat Soil Transmitted Helminth (STH) infections
m are target group.
2. Children below the age of 1 years
-
y
y
Which of the statements Ro
given above is/are correct?
A. 1 only n j o
B. 2 r it u
only
C.MBoth 1 and 2
D. Neither 1 nor 2

Your Answer :
Correct Answer : A

Answer Justification :

National Deworming Day

To combat Soil Transmitted Helminth (STH) infections, the Ministry has adopted a single
day strategy called National Deworming Day (NDD) wherein single dose of albendazole is
administered to children from 1-19 years of age group schools and anganwadi centres.
Hence statement 2 is incorrect. Till February 2017, 27.8 crore children were administered
albendazole.

prelims.insightsonindia.com 59
© Insights Active Learning | All rights reserved - 156493. You may not reproduce, distribute or exploit the contents in any form without
written permission by copyright owner. Copyright infringers may face civil and criminal liability
Total Marks : 200
Online Prelims TEST - 25 (SUBJECT)
( InsightsIAS Mock Test Series for UPSC Preliminary Exam 2020 ) Mark Scored : 31.33

91 The economic Survey introduces the idea of “trust as a public good that gets enhanced with greater
use”. Trust can be characterized as
1. The citizens can enjoy its benefits at no explicit financial cost.
2. The marginal cost of supplying this public good to an extra citizen is zero
3. Collective supply for all citizens means that it cannot be rejected.

Which of the statements given above is/are correct?


A. 1 only
B. 2 and 3 only
C. 1 and 2 only
D. 1, 2 and 3

Your Answer :
Correct Answer : D

Answer Justification :
1 3
The Survey introduces the idea of “trust as a public good that gets enhanced 27
5 with greater use”.
8 2
3 Hence, all the above
Trust can be conceptualized as a public good with the characteristics of:
6
statements are correct. - 8
om
l.cno explicit financial cost.
• Non-excludability: The citizens can enjoy its benefits at
ai
• Non-rival consumption: The marginal cost of@ g m
supplying this public good to an extra citizen is zero.
5
It is also
2 60
1
5for
r
• Non-rejectable: Collective supply7 all citizens means that it cannot be rejected.

y -m
92 Consider the followingR
o
statements
y
jolargest entrepreneurship ecosystem in the world ahead of the countries such as
1. India has the n3rd
u
itSouth Korea.
r
Brazil and
M
2. New firm creation has gone up dramatically in India since 2014.

Which of the statements given above is/are correct?


A. 1 only
B. 2 only
C. Both 1 and 2
D. Neither 1 nor 2

Your Answer : B
Correct Answer : C

Answer Justification :

India has the 3rd largest entrepreneurship ecosystem in the world ahead of the countries such as
Brazil and South Korea. Hence Statement 1 is correct.

New firm creation has gone up dramatically in India since 2014. As a result, 1, 24,000 new firms

prelims.insightsonindia.com 60
© Insights Active Learning | All rights reserved - 156493. You may not reproduce, distribute or exploit the contents in any form without
written permission by copyright owner. Copyright infringers may face civil and criminal liability
Total Marks : 200
Online Prelims TEST - 25 (SUBJECT)
( InsightsIAS Mock Test Series for UPSC Preliminary Exam 2020 ) Mark Scored : 31.33

were created in 2018 up from 70,000 in 2014. This growth is particularly pronounced for the
services sector reflecting India’s new economic structure. Hence Statement 2 is correct.

93 Consider the following statements regarding Essential Commodities Act (ECA)


1. The Act provides for the regulation and control of production, distribution and pricing of
commodities which are declared as essential.
2. It aims to ensure affordability of the essential commodities for the poor by restricting hoarding.
3. The Act makes distinguish between firms that genuinely need to hold stocks owing to nature of their
operations and firms that might speculatively hold stocks.

Which of the statements given above is/are correct?


A. 1 and 2 only
B. 2 only
C. 3 only
D. 1, 2 and 3
1 3
Your Answer : A 5 27
2
Correct Answer : A
6 38
-8
Answer Justification :
o m
il.c
The Essential Commodities Act, 1955 was enacted toaensure the easy availability of essential
commodities to consumers and to protect them from
@ gm exploitation by unscrupulous traders. The Act
provides for the regulation and control of 0 5
production, distribution and pricing of commodities which
6
12
are declared as essential. Hence Statement
5
1 is correct.
7
mr
- affordability of the essential commodities for the poor by restricting
The ECA, 1955 aims to ensure
o y
hoarding. To enable this, ECA controls the production, supply and distribution of, and trade and
R
y goods which are treated as essential commodities. Hence Statement 2 is
commerce in certain
correct. n j o
rit u
M
The Act does not distinguish between firms that genuinely need to hold stocks owing to nature of
their operations and firms that might speculatively hold stocks. Hence Statement 3 is incorrect.

94 Consider the following statements regarding Price Stabilization Fund


1. It is Centrally Sponsored Scheme
2. It aims to support market interventions for price control of perishable agri-horticultural
commodities
3. Under this, Procurement of the commodities will be undertaken directly from farmers or farmers’
organizations at farm gate/mandi and made available at a more reasonable price to the consumers.

Which of the statements given above is/are correct?


A. 3 only
B. 2 and 3 only
C. 1 and 2 only
D. 1, 2 and 3

prelims.insightsonindia.com 61
© Insights Active Learning | All rights reserved - 156493. You may not reproduce, distribute or exploit the contents in any form without
written permission by copyright owner. Copyright infringers may face civil and criminal liability
Total Marks : 200
Online Prelims TEST - 25 (SUBJECT)
( InsightsIAS Mock Test Series for UPSC Preliminary Exam 2020 ) Mark Scored : 31.33

Your Answer :
Correct Answer : B

Answer Justification :

Price Stabilization Fund (PSF) refers to any fund constituted for the purpose of containing extreme
volatility in prices of selected commodities. The amount in the fund is generally utilised for activities
aimed at bringing down/up the high/low prices say for instance, procurement of such products and
distribution of the same as and when required, so that prices remain in a range. It is Central
Sector Scheme. Hence Statement 1 is incorrect.

It aims to support market interventions for price control of perishable agri-horticultural


commodities PSF will be used to advance interest free loan to State Governments and Central
agencies to support their working capital and other expenses on procurement and distribution
interventions for such commodities. Hence Statement 2 is correct.

7 1to3theorconsumers.
Under this, Procurement of the commodities will be undertaken directly from farmers farmers’

52
organizations at farm gate/mandi and made available at a more reasonable price
Hence Statement 3 is correct. 8 2
6 3
-8
95 Consider the following statements regarding Network Products
.c om
1. Network Products exports accounts for 10% in India’sa l
iexport basket
2. The main category of Network Products exported gmby India is Road vehicles
@
5 only one with trade deficit in Network Products.
3. Among the major Asian countries, India is the
6 0
12
75 is/are correct?
Which of the statements given above
r
A. 1 and 2 only
B. 2 and 3 only y -m
o
C. 1, 2 and 3 y R
D. 3 only nj
o
rit u
M
Your Answer :
Correct Answer : C

Answer Justification :

The economic survey highlighted that in 2018, NP exports accounts for 10% in India’s export
basket, while these products account for about 50% of the total national exports of China, Japan and
Korea. Hence Statement 1 is correct.

Among the major Asian countries, India is the only one with trade deficit in NP. Hence Statement
3 is correct.

The main category of NP exported by India is Road vehicles with a share of 4.9 per cent in its total
exports in 2018. Hence Statement 2 is correct.

96 Consider the following statements regarding Authorised Economic Operator (AEO)

prelims.insightsonindia.com 62
© Insights Active Learning | All rights reserved - 156493. You may not reproduce, distribute or exploit the contents in any form without
written permission by copyright owner. Copyright infringers may face civil and criminal liability
Total Marks : 200
Online Prelims TEST - 25 (SUBJECT)
( InsightsIAS Mock Test Series for UPSC Preliminary Exam 2020 ) Mark Scored : 31.33

1. It is a programme under the aegis of the World Customs Organization (WCO) SAFE Framework of
Standards to secure and facilitate Global Trade.
2. It is a voluntary programme
3. It aims to enhance international supply chain security and facilitate movement of goods.

Which of the statements given above is/are correct?


A. 1 and 2 only
B. 2 and 3 only
C. 3 only
D. 1, 2 and 3

Your Answer :
Correct Answer : D

Answer Justification :

13Framework of
It is a programme under the aegis of the World Customs Organization (WCO) SAFE
7
52
Standards to secure and facilitate Global Trade. Hence Statement 1 is correct.
3 82
It is a voluntary programme and it aims to enhance international supply
- 86 chain security and facilitate
om
movement of goods. Hence Statement 2 and 3 are correct.
.c
il trade is approved by Customs as
Under this programme, an entity engaged in international a
compliant with supply chain security standards and
@ gmgranted AEO status. That entity is then
5
considered a ‘secure’ trader and a reliable0trading partner.
6
5 12
97 Consider the following statementsm r7
1. The absolute prices of o ay
-
vegetarian Thali have decreased since 2015-16

j oy R Thalis has improved over the time period from 2006-07 to 2019-20 by
2. Affordability of vegetarian
29%.
un
3. There israitspecific trend in variability of Thali prices at the All-India level.
M
Which of the statements given above is/are correct?
A. 1 and 2 only
B. 2 and 3 only
C. 1 and 3 only
D. 1, 2 and 3.

Your Answer : A
Correct Answer : A

Answer Justification :

The absolute prices of a vegetarian Thali have decreased since 2015-16 though it increased during
2019. This is true both across the country and regions i.e. North, South, East and West. Hence
Statement 1 is correct.

Affordability of vegetarian Thalis has improved over the time period from 2006-07 to 2019-20 by

prelims.insightsonindia.com 63
© Insights Active Learning | All rights reserved - 156493. You may not reproduce, distribute or exploit the contents in any form without
written permission by copyright owner. Copyright infringers may face civil and criminal liability
Total Marks : 200
Online Prelims TEST - 25 (SUBJECT)
( InsightsIAS Mock Test Series for UPSC Preliminary Exam 2020 ) Mark Scored : 31.33

29% and that for non-vegetarian Thalis by 18%. The average yearly gain to the household of 5
individuals would be around Rs. 10,887 and Rs. 11,787 for vegetarian and non-vegetarian Thali
respectively. Hence Statement 2 is correct.

There is no specific trend in variability of Thali prices at the All-India level. This is true across
regions and states and also over time. Hence Statement 3 is incorrect.

98 Consider the following statements


1. Over the last few years, the contribution of Non-debt Capital receipts has improved in the total pool
of Non-debt receipts.
2. The receipts from recovery of loans and advances have been declining over the years.

Which of the statements given above is/are correct?


A. 1 only
B. 2 only
C. Both 1 and 2
1 3
D. Neither 1 nor 2
5 27
2
6 38
-8
Your Answer :
Correct Answer : C
o m
il.c
Answer Justification :
gma
5
The major component of Non-debt Capital0receipts
@
is disinvestment Receipts that accrue to the
6
5 12
government on sale of public sector enterprises owned by the government (including sale of
7 at mobilising 1.05 lakh crore on account of disinvestment
strategic assets). Government aimed
r
proceeds asper 2019-20 BE.
y -mOver the last few years, the contribution of Non-debt Capital receipts
o
has improved in the total pool of Non-debt receipts. Hence Statement 1 is correct.

j oyR
n recovery of loans and advances have been declining over the years. Hence
The receipts from
u
t
i 2 is correct.
Mr
Statement

99 Consider the following statements regarding Merchandise Exports


1. Over the years, the merchandise exports to GDP ratio has been declining.
2. Petroleum, Oil and Lubricants (POL) exports have a dominant share in India’s export basket.
3. Growth in Non-POL exports drastically increased from 2009-14 to 2014-19.

Which of the statements given above is/are correct?


A. 3 only
B. 2 and 3 only
C. 1 and 2 only
D. 1, 2 and 3

Your Answer : D
Correct Answer : C

Answer Justification :

prelims.insightsonindia.com 64
© Insights Active Learning | All rights reserved - 156493. You may not reproduce, distribute or exploit the contents in any form without
written permission by copyright owner. Copyright infringers may face civil and criminal liability
Total Marks : 200
Online Prelims TEST - 25 (SUBJECT)
( InsightsIAS Mock Test Series for UPSC Preliminary Exam 2020 ) Mark Scored : 31.33

Over the years, the merchandise exports to GDP ratio has been declining. Slowdown of world output
(due to escalation of global trade tensions) and appreciation in the real exchange rate have
contributed to the declining exports to GDP ratio in recent times. Hence Statement 1 is correct.

Petroleum, Oil and Lubricants (POL) exports have a dominant share in India’s export basket. Hence
Statement 2 is correct.

Growth in Non-POL exports dropped significantly from 2009-14 to 2014-19. Hence Statement 3 is
incorrect.

100 Consider the following statements regarding performance of Non-Banking Financial Sector (NBFC)
in the recent past.
1. The growth of loans from NBFCs declined but the balance sheet of the NBFC sector grew
significantly.
2. Both the gross NPAs ratio and net NPAs ratio of NBFC sector increased.

1 3
Which of the statements given above is/are correct?
5 27
A. 1 only 2
B. 2 only 6 38
C. Both 1 and 2 -8
o m
il.c
D. Neither 1 nor 2

Your Answer : B gma


5 @
Correct Answer : C 0
126
5
Answer Justification :
r7
-m
oy NBFCs declined but the balance sheet of the NBFC sector grew
The growth of loans from
R
y
significantly. The sector also witnessed liquidity stress. Hence Statement 1 is correct.

u njo
it to risk weighted assets ratio (CRAR) of NBFC sector remained at 19.5 per cent as
The Capital
r
M the regulatory requirement of 15 per cent. Both the gross NPAs ratio and net NPAs ratio of
against
NBFC sector increased. The Return on Assets (RoA) and the Return on Equity (RoE) has declined in
the previous year. Hence Statement 2 is correct.

prelims.insightsonindia.com 65
© Insights Active Learning | All rights reserved - 156493. You may not reproduce, distribute or exploit the contents in any form without
written permission by copyright owner. Copyright infringers may face civil and criminal liability

You might also like